Uworld

Pataasin ang iyong marka sa homework at exams ngayon gamit ang Quizwiz!

__ are the most common cause of encephalitis outbreaks in the United States.

Arboviruses, small RNA viruses transmitted by biting arthropods (west nile, etc) Because no vaccines are currently available, prevention primarily involves eliminating the vector arthropods (eg, infected mosquitos).

an abnormal connection between arteries and veins bypassing capillaries which develops due to disrupted angiogenesis

Arteriovenous malformation Cerebral arteriovenous malformations (AVMs) are vascular malformations histologically characterized by a tangle of abnormal vessels, including large veins with thickened walls and irregular arteries. AVMs may present with intracranial hemorrhage, headache, and/or seizures. Radiologic findings include abnormal vasculature with a "bag of worms" appearance and multiple dark flow voids.

Rupture of what part of bladder causes INTRAperitoneal fluid accumulation

Bladder dome (bc while bladder is exztraperitoneal, dome extends into peritoneal cavity when filled)

Hemorrhagic Fever Viruses

Bunyaviruses • Rift Valley Fever virus (mosquito) • Crimean-Congo hemorrhagic fever virus (tick) • Hanta virus (rodents) Filoviruses • Ebola/Marburg Arenaviruses • Lassa fever (rats) Sometimes flaviviruses • Yellow fever, dengue can progress

Transected nerves undergo wallerian degeneration, which involves axonal dissolution while activated Schwann cells and macrophages degrade myelin. In addition to clearing cellular debris, Schwann cells and macrophages stimulate formation of a growth cone from the proximal axon stump to facilitate nerve regeneration. However, if the regenerating axons cannot find their distal target, regeneration can result in formation of a disorganized mass of axonal processes, Schwann cells, vascular hyalinization, and fibrosis known as a neuroma.

Chronic axonal injury or inflammation (such as occurs with a neuroma) induces increased expression of voltage-gated sodium channels, which are important for the generation and propagation of action potentials. The density of voltage-gated sodium channels correlates with the excitability of nerves; therefore, the upregulated sodium channels in injured nerves and neuromas increase nociceptive (pain) sensitivity. To support the diagnosis of a neuroma, local anesthetic, which blocks sodium channels in the neuronal cell membrane, is administered to prevent depolarization of the nerve. This often provides immediate relief of pain

Duputyrun contracture is a slowly progressive fibroproliferative disease of the palmar fascia that results in contractures that draw the 4th and 5th fingers into flexion

Nodules form on the fascia, which cause the contractures

Dihydrobiopterin reductase deficiency results in impaired tetrahydrobiopterin (BH4) levels. BH4 is an important cofactor for both phenylalanine hydroxylase and tyrosine hydroxylase. Although phenylalanine levels can be corrected with dietary restriction, downstream deficiencies of dopamine, norepinephrine, epinephrine, and serotonin lead to progressive neurologic deterioration.

Normally, dopamine from the tuberoinfundibular system tonically inhibits prolactin release. Decreased BH4 causes lower levels of dopamine, resulting in increased prolactin levels

lacrimation and yawning is seen in what withdrawal

Opioid Opioid withdrawal can start as soon as 6-12 hours after cessation of a short-acting opioid such as heroin and usually peaks within 24-48 hours. Symptoms include nausea, vomiting, diarrhea, abdominal cramping, and myalgia. It is generally nonlife-threatening and vital signs are usually normal. Dilated pupils, prominent piloerection, and hyperactive bowel sounds are seen on examination.

Janeway lesions are nontender, macular, and erythematous lesions typically located on the palms and soles of patients with acute infective endocarditis and are the result of septic embolization from valvular vegetations.

Osler nodes are tender, violaceous nodules typically located in the pulp of fingers and toes. The pathogenesis of Osler nodes is immune-complex deposition in the skin. Roth spots: edematous & hemorrhagic lesions of the retina

Spongiosa filling medullary canals with no mature trabeculae describes

Osteopetrosis ("marble bone disease") is characterized by persistence of the primary spongiosa in the medullary cavity with no mature trabeculae. It is caused by decreased osteoclastic bone resorption, resulting in accumulation of woven bone and diffuse skeletal thickening.

Drug reaction with eosinophilia and systemic symptoms (DRESS) syndrome typically occurs 2-8 weeks after exposure to high-risk drugs such as anticonvulsants (eg, phenytoin, carbamazepine), allopurinol, sulfonamides (eg, sulfasalazine), and antibiotics (eg, minocycline, vancomycin). Patients typically develop fever, generalized lymphadenopathy, facial edema, diffuse skin rash, eosinophilia, and internal organ dysfunction.

Other affected organs can include the liver (hepatomegaly, jaundice), kidney (acute interstitial nephritis), and lung (cough, dyspnea). Laboratory studies usually show eosinophilia, atypical lymphocytosis, and elevated serum alanine transaminase. Clinical findings improve over several weeks following withdrawal of the drug.

Osteoblasts synthesize bone matrix and express alkaline phosphatase (AlkP), which promotes normal bone mineralization by increasing local concentrations of inorganic phosphorus. Serum AlkP levels correlate with osteoblastic activity

Other markers of osteoblast activity include N-terminal propeptide of type 1 procollagen, which is released during post-translation cleavage of type 1 procollagen.

_is the most common cause of death in infants born with congenital diaphragmatic hernia

Pulmonary hypoplasia due to impaired growth and inflation of the newborn's lungs. Oligohydramnios can also cause pulmonary hypoplasia.

Internal hemorrhoids originate above the dentate line and are covered by columnar epithelium. They have autonomic innervation from the inferior hypogastric plexus, which is only sensitive to stretch and not pain, temperature, or touch.

External hemorrhoids, which originate below the dentate line, are covered by modified squamous epithelium and have cutaneous (somatic) nervous innervation from the inferior rectal nerve, a branch of the pudendal nerve.

Adverse effects associated with the excessive use of anabolic steroids include acne, gynecomastia, azoospermia, decreased testicular size and increased aggression. Hypertension, dyslipidemia, cholestatic hepatitis and hepatic failure may also occur.

Serum testosterone levels can be low in individuals taking only synthetic androgens (eg, trenbolone), but are often within the normal range or elevated due to exogenous testosterone intake. Although serum testosterone may appear adequate, lower than normal local testosterone levels in the seminiferous tubules lead to decreased spermatogenesis.

What are benign, fluid-filled masses that occur due to mucoid degeneration of periarticular tissue.

Ganglion cysts They present as painless, smooth, rubbery, round structures that overlie joints (eg, wrist) or tendons and transilluminate with a penlight. Most resolve spontaneously without intervention. Mucinous fluid collects via a 1-way valve mechanism

Fluid filled mass on a joint that transilluminates

Ganglion cysts Mucinous fluid collects via a 1-way valve mechanism They present as painless, smooth, rubbery, round structures that overlie joints (eg, wrist) or tendons and transilluminate with a penlight. Most resolve spontaneously without intervention.

One of the most dangerous effects of SHS exposure (pre- and postnatal) is the increased risk of sudden infant death syndrome (SIDS). SIDS refers to the unexpected death of a seemingly healthy infant during sleep. Up to half of all SIDS cases are due to SHS exposure, likely due to impaired arousal and abnormal cardiovascular responses to stimuli.

SHS also increases the risk of recurrent otitis media, asthma, and other respiratory tract illnesses (eg, pneumonia) in children

What are the dialyzable substances?

Salicylates Lithium Isopropyl alcohol Magnesium laxatives Ethylene glycol

Unlike epinephrine, glucagon has an insignificant effect on glucose homeostasis in the skeletal muscle, adipose tissue, and renal cortex.

Glucagon increases serum glucose by increasing hepatic glycogenolysis and gluconeogenesis. Glucagon also stimulates insulin secretion from the pancreas

Fluphenazine MOA

High potency antipsychotic

medications that inhibit neprilysin (eg, sacubitril) lead to increased levels of ANP and BNP and promote beneficial effects in heart failure

However, because neprilysin is also responsible for inactivating angiotensin II, inhibition of neprilysin further stimulates deleterious vasoconstriction and fluid retention via increased angiotensin II levels. Therefore, in treating heart failure, neprilysin inhibitors are combined with an angiotensin II-receptor blocker (eg, sacubitril-valsartan) to mitigate these negative effects

The amphetamine lisdexamfetamine has been used to treat binge eating disorder, but not bulimia nervosa.

Hyperproteinemia causes increased glomerular capillary oncotic pressure thereby decreasing GFR. This will have no effect on the RPF, but will cause a decreased FF due to the decreased GFR. (Choice B) Bladder neck obstruction causes an increase in Bowman's space hydrostatic pressure thereby decreasing GFR. This too will have no effect on the RPF, but will cause a decreased FF due to the decreased GFR

Why is Gingival hyperplasia is a common side effect of phenytoin

Phenytoin causes increased expression of platelet-derived growth factor (PDGF). When gingival macrophages are exposed to increased amounts of PDGF, they stimulate proliferation of gingival cells and alveolar bone sometimes reversible when phenytoin is withdrawn.

immunohistochemistry is positive for synaptophysin, chromogranin, and neuron-specific enolase.

Pheochromocytoma

Inhaled anesthetics, such as halothane, can be associated with a highly lethal fulminant hepatitis that cannot be distinguished histologically from acute viral hepatitis. Patients have significantly elevated aminotransferase levels due to massive hepatocellular injury and a prolonged prothrombin time due to failure of hepatic synthetic function.

Laboratory findings typically include markedly elevated serum aminotransferase levels, prolonged prothrombin time, leukocytosis, and eosinophilia. The prolonged prothrombin time is due to failure of hepatic synthetic function and deficiency of factor VII (which has the shortest half-life of all the procoagulant factors).

Acute kidney injury in rhabdomyolysis occurs due to myoglobin degradation and heme pigment release. Heme pigment causes acute tubular necrosis (ATN) through direct cytotoxicity and renal vasoconstriction (ie, ischemia). In ATN, injured tubular epithelial cells slough off into the tubular lumen, forming granular, muddy brown casts. Heme pigment in myoglobin cross-reacts with the urine dipstick reagent that detects hemoglobin, leading to a false-positive result for blood in urine; however, microscopy shows no red blood cells (RBCs).

Laboratory studies often show markedly elevated creatine kinase levels and acute kidney injury with electrolyte disturbances (eg, hyperkalemia, hyperphosphatemia, hypocalcemia, metabolic acidosis).

_ presents with inspiratory stridor during infancy due to collapse of supraglottic tissues during inspiration

Laryngomalacia stridor is classically worse in supine position, improves with upright position

B anthracis:

Lethal factor is a zinc metalloproteinase that inhibits MAP kinase signal transduction, leading to cell death. Edema factor is a calmodulin-dependent adenylate cyclase that increases cyclic AMP concentration, leading to accumulation of fluid within/between cells and suppression of neutrophil and macrophage functio

lace-like purplish discoloration of the skin over his arms, shoulders, and legs think ___ from __ Parkinson drug

Livedo reticularis from Amantadine

Pleural biopsy of what lung cancer shows proliferation of epithelioid-type cells that are joined by desmosomes, contain abundant tonofilaments, and are studded with very long microvilli.

Mesothelioma

Methadone is a full mu-opioid receptor agonist used for withdrawal and maintenance treatment for opioid use disorder. It has a long half-life, which allows it to effectively suppress cravings and withdrawal symptoms; it also blocks the euphoric effects of other opioids by maintaining high tolerance levels (Choice B). Adverse effects of methadone include QT interval prolongation and respiratory depression, accounting for its lethality in overdose.

Methadone, like other opioids, produces dependence, and abrupt discontinuation of methadone causes withdrawal symptoms. The symptoms of methadone withdrawal are more prolonged and less severe than those of heroin due to methadone's long half-life but are still present

Coronary sinus has the most deoxygenated blood of anywhere

Myocardial oxygen extraction exceeds that of any other tissue or organ; therefore, the cardiac venous blood in the coronary sinus is the most deoxygenated blood in the body. Due to the high degree of oxygen extraction, increases in myocardial oxygen demand can only be met by an increase in coronary blood flow

What occurs to the stenotic kidney in unilateral RAS

Narrowing of the renal artery in the stenotic kidney leads to hypoperfusion and renal parenchymal ischemia, which manifests as diffuse cortical thinning, tubular atrophy, interstitial fibrosis, and small crowded glomeruli. Juxtaglomerular apparatus enlargement can also occur due to chronic stimulation and increased renin release.

Cricothyrotomy is indicated when an emergency airway is required and orotracheal or nasotracheal intubation is either unsuccessful or contraindicated (eg, massive hemorrhage, vomiting, facial trauma, airway obstruction). The procedure establishes an airway through the placement of a tube between the cricoid and thyroid cartilages and requires incision through the following structures:

Skin Superficial cervical fascia (including subcutaneous fat and platysma muscle) Investing and pretracheal layers of the deep cervical fascia Cricothyroid membrane

Septic abortion refers to any type of abortion resulting in infected retained products of conception in the uterine cavity

Staphylococcus aureus is a very common pathogen in septic abortion; other causative organisms include gram-negative bacilli (eg, Escherichia coli) and group B Streptococcus. These organisms are part of normal vaginal flora and seed the uterine cavity during instrumentation, thereafter proliferating in the retained tissue. Most infections associated with septic abortion are confined to the placental tissue, but patients can become septic if toxin-producing bacteria gain access to the intervillous space and bloodstream. Septic abortion typically presents with fever, abdominal pain, uterine tenderness, and/or foul-smelling discharge after pregnancy termination

Somatostatinoma triad of

Steatorrhea (decrease secretin) Gallstones (decrease CCK) Diabetes (hyperglycemia/decrease insulin) Also achlorhydria (decrease gastrin)

Clostridium difficile requires additional contact precautions, including handwashing with soap and water, a gown for any patient contact, and nonsterile gloves that should be changed after contact with contaminated secretions. Alcohol-based hand sanitizers do not kill the bacterial spores.

Sterile gloves generally are used for minor procedures or surgeries but are not required in patients with contact precautions.

bisacodyl MOA

Stimulant laxative (eg, bisacodyl, senna) Activates enteric nerves in myenteric plexus to stimulate peristalsis Side effects: abdominal cramping, electrolyte disturbances

Aspergillus is a ubiquitous fungal organism whose reproductive forms classically exhibit a "broom-like" structure on silver staining . Patients with a history of tuberculosis are prone to development of aspergillomas ("fungus balls") in lung cavities

These organisms have a characteristic microscopic morphology, with septate hyphae out of which asexual fruiting structures known as conidiophores project. Conidiophores each have a terminal vesicle with outwardly radiating phialides and conidiospores attached, giving a "broom-like" appearance.

Treatment for neonatal abstinence syndrome

The treatment of choice for acute opioid withdrawal in neonates is opiate replacement, usually morphine or methadone. The dose is gradually increased until symptoms are controlled, and then the patient is weaned off over several weeks.

One complication of long standing rheumatoid arthritis is {{c1::subluxation}} of the {{c2::cervical}} spine

This is a potential lethal feature of RA - erosions in C1/C2 can occur

an elastic protein that anchors the beta-myosin heavy chain to the Z-discs and likely contributes to passive myocardial tension

Titin absence of complete titin proteins leads to myocardial dysfunction (familial dilated cardiomyopathy)

Pulmonary arterial hypertension: The pathogenesis involves thickening of the smooth muscle medial layer and hyperplasia of the intimal layer of small pulmonary arteries and arterioles. Progressive intimal fibrosis takes place in a concentric, "onion skin" form with eventual development of plexiform lesions. Vascular resistance in the pulmonary arterial system is markedly increased, leading to elevated pulmonary arterial pressure.

To compensate for elevated pulmonary arterial pressure, right ventricular hypertrophy develops, often evidenced by right ventricular heave (detected by left parasternal lift) on physical examination. Over time, right-sided heart failure can occur with jugular venous distension, hepatomegaly, and peripheral edema.

The falciform ligament attaches the liver to the anterior body wall. It is a derivative of the embryonic ventral mesentery and contains the round ligament, the remnant of the fetal umbilical vein.

Tryptase is an enzyme that is relatively specific to mast cells, and elevated serum levels of tryptase are often used to support a clinical diagnosis of anaphylaxis after the patient has been stabilized

is the major amino acid in the blood because it transports excess ammonia from peripheral tissues to the kidney.

_Glutamine In the nephron, the amide nitrogen is hydrolyzed by glutaminase to regenerate glutamate and a free ammonium ion, which can then be excreted in the urine.

The clinical manifestations of schistosomiasis result from _ directed against the eggs that is composed of _

a TH2-mediated granulomatous response Composed of infiltrating TH2 cells, eosinophils, and M2 macrophages. This ultimately leads to the development of marked fibrosis and ulceration and scarring of the bowel or bladder/ureters (depending on the species). Eggs that settle into the presinusoidal radicals of the portal vein can cause periportal "pipestem" fibrosis (pathognomonic for hepatic schistosomiasis).

Painful neuropathy is treated with serotonin-norepinephrine reuptake inhibitors, gabapentinoids, or tricyclic antidepressants.

diabetes mellitus

Chronically elevated venous pressure in the lower extremities can lead to incompetent venous valves and venous dilation (varicose veins). Venous congestion and tissue ischemia can result in venous stasis dermatitis

dilation of the veins (varicose veins) Extravasation of red blood cells into the tissues leads to iron deposition and the characteristic brawny discoloration. Common risk factors for varicose veins and stasis dermatitis include obstruction of venous return (eg, obesity, pregnancy) and conditions that damage the venous valves (eg, deep venous thrombosis). Prolonged standing and sedentary lifestyles are also associated with varicose veins.

Food caught in the throat lodges in the

piriform recess Nerve just deep to the mucosa of the piriform recess susceptible to piercing by sharp ingested objects: superior laryngeal n. (Branch of vagus)

The duration of action of Succinylcholine is determined by its metabolism by

plasma cholinesterase Aka pseudocholinesterase (butyrylcholinesterase), made by the liver and encoded by the BCHE gene; duration of of action is typically <10 minutes patients with homozygous BCHE mutations result in delayed metabolism of succinylcholine, mivacurium, heroin, and cocaine

Why see Hemosiderin-laden macrophages located in the lungs in heart failure

usually the result of chronic passive lung congestion in the setting of heart failure (eg, left ventricular systolic dysfunction). Elevated pulmonary venous pressure leads to transudation of fluid across the alveolar-capillary membrane (pulmonary edema) and can cause breaks in the endothelium with extravasation of red blood cells into the alveoli and lung parenchyma. As alveolar macrophages engulf and degrade the extravasated red blood cells, the released iron accumulates as intracellular hemosiderin.

NK cells:

Do not require the thymus for maturation and are present in athymic patients

Dopamine agonists have a chemical structure similar to the neurotransmitter dopamine and directly stimulate dopamine receptors. The 2 classes of dopamine agonists are ergot compounds (eg, bromocriptine) and nonergot compounds (eg, pramipexole, ropinirole).

Dopamine agonists have an important role in the treatment of Parkinson disease (PD) as these medications have a long half-life and can delay the need to start levodopa, thereby postponing the development of motor fluctuations until later in the disease course. Bromocriptine also treats hyperprolactinemia.

branches of the vagus nerve mnemonic:

Ebbys Very Beautiful Sisters Maria And Clare Like Pooping! Esophageal, vagal, bronchial, superior laryngeal (internal: sensory, external: motor), meningeal & auricular, cardiac, recurrent Laryngeal, pharyngeal

First-line treatment of Wilson disease is with penicillamine, a copper chelating agent. Penicillamine binds free copper as well as reduces copper bound to other proteins. This reduction preferentially increases affinity of copper to the chelating agent. The chelator-copper complex is soluble, thereby increasing urinary copper excretion.

(Choice A) Zinc decreases intestinal absorption of copper and can be used in conjunction with penicillamine to treat Wilson disease.

Describe euthyroid sick syndrome (ESS)

(also known as low T3 syndrome), characterized by a low T3 level, normal thyroid-stimulating hormone (TSH), and normal T4. Thyroid hormone is released primarily in the form of T4, with conversion to T3 (the more active form) by 5'-deiodinase (types I and II) in peripheral tissues. In severe illness, high levels of cortisol, inflammatory cytokines, and free fatty acids suppress deiodination of T4, resulting in lower circulating levels of T3. Concurrently, upregulation of 5'-deiodinase type III leads to increased conversion of T4 to reverse T3 (inactive form). ESS is thought to be a form of mild, transient central hypothyroidism, intended to decrease maladaptive catabolism in severe illness; treatment with exogenous thyroid hormone does not improve outcomes and is not indicated. With increasing duration and severity of illness, central production of thyrotropin-releasing hormone (TRH) and TSH are also suppressed, leading to lower circulating TSH and T4 levels.

Gallstone ileus results from passage of a large gallstone through a cholecystenteric fistula into the small bowel, where it ultimately causes obstruction at the ileum. Patients typically present with symptoms/signs of small bowel obstruction, and an abdominal x-ray may reveal gas within the gallbladder and biliary tree.

(pneumobilia).

Hereditary pancreatitis may be caused by mutations in the {{c2::cleavage}} site of trypsinogen or the {{c1::SPINK1}} protein

- SPINK1 - serine peptidase inhibitor kazal-type1 is secreted by pancreatic acinar cells and functions as a trypsin inhibitor, impeding the activity of trypsinogen molecules that become premature activated within the pancreas - most common mutation leads to production of abnormal trypsin that is not susceptible to inactivating cleavage by trypsin

DKA presents with hypophosphosphatemia

Acidosis can cause the phosphate to shift to the extracellular fluid, and is lost in urine via osmotic diuresis

The ureter can be injured during hysterectomy due to its close proximity to the uterine structures. The distal ureter may be severed during ligation of the uterine vessels because the ureter passes inferior and lateral to the uterine artery at the level of the internal cervical os prior to entering the bladder (eg, "water under the bridge")

A ureteral obstruction injury with a suture leads to flank pain due to distention of the proximal ureter with urine. A transection injury of the ureter causes localized leakage of urine as well as flank pain. Since the second ureter is uninjured, patients are able to void (urinate) normally. Bilateral ureteral injury is extremely unlikely and would result in anuria and bilateral flank pain

Leukocidin

A virulence factor of certain bacteria (e.g., S. aureus). Creates pores in cell membranes, which results in necrotic mucocutaneous lesions. The most virulent strains of S aureus have been infected with a bacteriophage that transmits genes for Panton-Valentine leukocidin (PVL), a cytotoxin that destroys leukocytes and causes tissue necrosis. PVL is primarily seen in community-acquired strains of S aureus that are methicillin-resistant (methicillin-resistance is conferred by a different mobile genetic element [mecA]). Staphylococcus aureus strains that express Panton-Valentine leukocidin (PVL), a protease that kills leukocytes and causes necrosis, are most likely to cause skin or soft-tissue abscess and invasive disease (eg, necrotizing pneumonia). PVL expression is most common in community-acquired, methicillin-resistant strains.

P-glycoprotein is an efflux pump found on brain capillary endothelial cells (part of the blood-brain barrier) that inhibits a wide range of substrates from entering the brain. Inhibition of p-glycoprotein can improve drug delivery to the CNS.

ATP dependent transporter

How long does renal compensation take at high altitude

About 72 hours At high altitude, the low partial pressure of inspired oxygen (PiO2) leads to hypoxemia that triggers hyperventilation with increased exhalation of CO2 and resulting respiratory alkalosis. The kidneys compensate by increasing bicarbonate (HCO3−) excretion to decrease serum HCO3− and help normalize pH. Hypoxemia persists, but physiologic adjustments take place to improve O2 use.

Although expiration is largely achieved through passive recoil, active expiration is aided by the internal intercostals (innervated by thoracic nerve rootlets) and abdominal muscles (innervated by thoracic and lumbar nerve roots).

Accessory muscles of inspiration elevate the sternum and ribs to further expand the thoracic cavity. These include the external intercostals (innervated by thoracic nerve roots), the sternocleidomastoid (innervated by C1-C4), and the scalenes (innervated by C4-C8).

What substance stimulates chromaffin cells/adrenal medulla

Acetylcholine

Vascular smooth muscle cells (VSMCs) are the only cells within the atherosclerotic plaque capable of synthesizing structurally important collagen isoforms and other matrix components. Progressive enlargement of the plaque results in remodeling of the extracellular matrix and VSMC death, promoting development of vulnerable plaques with an increased propensity for rupture

Activated macrophages and T lymphocytes secrete growth factors that recruit the VSMCs responsible for forming the fibrous cap, but they are not directly responsible for the dense deposition of fibrillar collagen. Macrophages also produce matrix metalloproteinases and tissue factors that degrade the extracellular matrix, causing the formation of a large, soft lipid-rich core with thinning of the fibrous cap. Such vulnerable plaques have an increased propensity for rupture.

Excess NADH inhibits free fatty acid oxidation, thereby diverting free fatty acids away from lipolysis to the formation of triglycerides. This contributes to hepatic steatosis in alcoholic patients but does not cause hypoglycemia

Alcohol does not inhibit glycogenolysis (Choice B), and so in the initial phase of binge drinking (heavy alcohol intake with reduced nutritional carbohydrate intake), hepatic glycogenolysis is able to maintain euglycemia. However, after a prolonged binge, hepatic glycogen is eventually depleted and blood glucose levels drop. Ethanol inhibits gluconeogenesis and can cause hypoglycemia once hepatic glycogen stores are depleted

Transcriptional repression (silencing) is one of the mechanisms by which mutated huntingtin is thought to cause disease.

Alteration of gene expression in Huntington disease occurs in part due to deacetylation of histones. This prevents the transcription of certain genes that code for neurotrophic factors, contributing to neuronal cell death

Prussian blue stain detects ferric iron stores (eg, ferritin and hemosiderin). In the Prussian blue reaction, colorless potassium ferrocyanide is converted by iron to blue-black ferric ferrocyanide. Macrophages containing golden-brown cytoplasmic granules that turn blue with Prussian blue staining represent hemosiderin-laden macrophages (siderophages). These cells may be found in any tissue where macrophages encounter extravasated red blood cells; in the alveolar parenchyma, they are often called "heart failure cells."

Alveolar hemosiderin-laden macrophages indicate alveolar hemorrhage. They most commonly result from chronic elevation of pulmonary capillary hydrostatic pressure in the setting of left-sided heart failure.

The adaptations of athlete's heart differ depending on whether intensive endurance training (eg, long-distance running) or resistance training (eg, heavy weightlifting) is pursued. With endurance training, systemic vascular resistance (SVR) is reduced (due to vasodilation in skeletal muscle) and blood volume is increased, leading to increased left ventricular (LV) volume load. To compensate, LV eccentric hypertrophy (ie, LV wall lengthening) develops, which increases LV cavity size to increase diastolic filling capacity and facilitate increased stroke volume and cardiac output. Right ventricular (RV) cavity enlargement also takes place to increase diastolic filling and accommodate the increase in cardiac output; because the right ventricle and left ventricle are arranged in series and contract at the same rate, RV stroke volume must increase to match LV stroke volume and prevent backup of blood flow.

An increase in stroke volume is maintained at rest and allows for adequate cardiac output at low resting heart rate (eg, <55/min). The anterior location of the pulmonic valve and slightly smaller valve area compared to the aortic valve often create a pulmonic flow murmur due to the high stroke volume. The murmur is best heard at the left second intercostal space in the supine position and may disappear when venous return is decreased with standing.

_ is an inflammatory condition characterized by simultaneous erosion of bone and new bone formation.

Ankylosing spondylitis Bone erosions occur primarily in vertebral bodies; however, new bone formation typically occurs at the junction of the periosteal margin and adjacent cartilage, leading to bridging syndesmophytes and ankylosis. This causes spinal rigidity, postural alterations, and increased risk of fracture

__ is the most common cause of familial dilated cardiomyopathy

Autosomal dominant mutations in the TTN gene, which encodes for the sarcomere protein titin TTN gene mutations follow autosomal dominant inheritance; however, they have incomplete penetrance, leading to delayed or absent clinical manifestations in some family members.

Chronic alcohol use downregulates inhibitory GABA receptors and upregulates excitatory NMDA glutamate receptors. Therefore, sudden cessation of alcohol leads to decreased GABA activity and increased glutamate activity, resulting in CNS overexcitation (eg, agitation, disorientation, tremulousness, autonomic hyperactivity).

Because warfarin inhibits gamma carboxylation of new vitamin K-dependent clotting factors, therapeutic efficacy is delayed until preexisting clotting factors in the plasma are consumed. Although INR tends to slowly increase in the first few days of administration due to the short half-life of factor VII (4-6 hours), full therapeutic effect does not typically occur for 3 days due to the long half-life of factor II. Warfarin also inhibits gamma carboxylation of the proteins C and S. Because these proteins exert an anticoagulant effect, the initiation of warfarin can be associated with an initial procoagulant state. However, once preexisting vitamin K-dependent clotting factors are consumed, an anticoagulant effect is established.

The Beers criteria identify drugs that should be used with caution in geriatric patients. Common drugs to avoid include anticholinergics (eg, first-generation antihistamines), centrally acting alpha-2 agonists, tricyclic antidepressants, benzodiazepines (and other sedating medications), antipsychotics, many antiarrhythmics (eg, digoxin), and skeletal muscle relaxants.

Branches of the subclavian artery can be remembered with the mnemonic (VIT CD) V - {{c1::Vertebral artery}} I - {{c2::Internal thoracic (internal mammary) artery}} T - {{c3::Thyrocervical trunk}} C - {{c4::Costocervical artery}} D - {{c5::Dorsal scapular artery}}

Patients with a long history of hemodialysis can also develop median nerve compression through deposition of β2-microglobulin (dialysis-associated amyloidosis) in the carpal tunnel.

CTS is associated with conditions that reduce carpal tunnel space, including pregnancy (fluid accumulation), hypothyroidism (glycosaminoglycan buildup), diabetes mellitus (connective tissue thickening), and rheumatoid arthritis (tendon inflammation).

What can be used to treat neuropathic pain by Loss of membrane potential in nociceptive fibers

Capsaicin cream (decreases substance P)

Contraindications to beta blockers include bradycardia or heart block, hypotension, and overt heart failure (eg, pulmonary edema). Also, non-cardioselective beta blockers (eg, propranolol, nadolol) can trigger bronchospasm in patients with underlying obstructive lung disease (asthma, chronic obstructive pulmonary disease [COPD]) due to beta-2 receptor blockade and should be avoided

Cardioselective beta blockers with predominant action on beta-1 receptors (eg, metoprolol, atenolol, bisoprolol, nebivolol) are safe in patients with stable obstructive lung disease and are the beta blocker of choice in these patients. Combined beta and alpha receptor blockers (eg, carvedilol, labetalol) are also well tolerated and have been used safely in patients with COPD

Zika virus infection in utero causes infection of neural progenitor cells, resulting in impaired fetal brain development. Due to the destruction of the neurons, patients with congenital Zika virus infection typically have microcephaly, intracranial calcifications (a sign of tissue necrosis), and ventriculomegaly

Causes conjunctivitis, low-grade pyrexia, and itchy rash in 20% of cases.

Renin-secreting tumors (reninomas) are rare, small, solitary, benign juxtaglomerular cell neoplasms. Reninomas should be strongly considered in patients with marked hyperreninemia and hypertension who clearly do not have renovascular disease.

Causes of secondary hyperaldosteronism include renal artery stenosis (typically associated with fibromuscular dysplasia or atherosclerosis), diuretic use, malignant hypertension (which leads to microvascular damage and renal ischemia), and renin-secreting tumors

Radiologic findings of __ include abnormal vasculature with a "bag of worms" appearance and multiple dark flow voids.

Cerebral arteriovenous malformations (AVMs) are vascular malformations histologically characterized by a tangle of abnormal vessels, including large veins with thickened walls and irregular arteries.

Class 1C antiarrhythmics such as flecainide are potent sodium channel blockers that have increased effect at faster heart rates (use-dependence). This makes them more effective at treating tachyarrhythmias, but can also cause prolonged QRS duration (a proarrhythmic effect) at higher heart rates

Class III agents demonstrate reverse use-dependence (the slower the heart rate, the more the QT interval is prolonged).

Scabies is a skin infection caused by the human itch mite, Sarcoptes scabiei. Transmission occurs from prolonged skin-to-skin contact with an infected individual but may occasionally occur due to contact with contaminated fomites (eg, bedsheets, clothing). Mites burrow under the epidermis and evoke a delayed-type hypersensitivity reaction to mite feces and eggs, which leads to an intensely pruritic rash. The mites can often be seen on microscopy.

Classic scabies is associated with a mild mite burden (10-15 mites). Patients usually have several small erythematous papules on the sides/webs of the fingers, wrists, elbows, axillae, waist, and genitalia. Excoriations and burrows (thin, serpiginous red tracks) may be seen. In contrast, patients with impaired cell-mediated immunity (eg, HIV) are unable to contain the infection and usually develop crusted scabies, which is associated with thousands or millions of mites. These patients often have mild pruritis (because the inflammatory response is muted) and several erythematous patches with scaling and crusting. Treatment is required to prevent discomfort, transmission, and potential complications (eg, secondary bacterial infection). First-line therapy includes topical permethrin, which blocks mite neurotransmission by impairing voltage-gated sodium channels. Oral ivermectin, an antiparasitic agent that binds chloride ion channels in invertebrate nerve and muscle cells, is an alternate medication for classic scabies that is used in combination with permethrin for crusted scabies.

Critical illness can lead to weakness due to both myopathy (eg, atrophy of muscles) and polyneuropathy (eg, axonal degeneration, decreased nerve excitability). This can lead to both extremity weakness and difficulty breathing due to chest wall weakness

Clinical features of CIM and CIP typically include symmetric weakness greater in proximal, rather than distal, muscles with decreased deep tendon reflexes. It is often diagnosed when patients are unable to be liberated from the ventilator due to chest wall weakness.

_ withdrawal is characterized by the development of acute depression (with suicidal ideation) accompanied by fatigue, hypersomnia, hyperphagia, and vivid dreams.

Cocaine In contrast to withdrawal from opiates, alcohol, and benzodiazepines, withdrawal from cocaine and other stimulants usually results in minor physical symptoms. No medication has proven to be effective in treating cocaine withdrawal, and treatment is supportive only.

Mitral valve prolapse with regurgitation is the most common predisposing condition for native valve infective endocarditis (IE) in developed nations. Rheumatic heart disease remains a frequent cause of IE in developing nations.

Congenital heart lesions that predispose to increased risk of IE include bicuspid aortic valve, ventricular septal defects, patent ductus arteriosus, and unrepaired tetralogy of Fallot

Dupuytren contracture (DC), a condition caused by progressive fibrosis of the superficial palmar fascia. The disease typically involves the fascia at the base of the middle, ring, and little fingers. Although the etiology of DC is unknown, risk factors include age >50, male sex, family history, and Northern European ancestry.

DC is thought to be caused by overstimulation of the Wnt-signaling pathway, which regulates cellular proliferation. Initial findings include painless fascial thickening with puckering of the skin just proximal to the affected metacarpophalangeal joint(s). As fibrosis continues, pathognomonic fascial nodules form along the flexor tendons composed of proliferating fibroblasts and disordered type III collagen. The nodules eventually coalesce into palpable fibrotic cords that tether the flexor tendon to the palmar fascia, leading to loss of finger extension (ie, contractures) at the metacarpophalangeal and proximal interphalangeal joints.

Damage above the red nucleus (eg, cerebral hemisphere, internal capsule) typically results in decorticate (flexor) posturing due to loss of descending inhibition of the red nucleus and subsequent hyperactivity of upper limb flexors (Choices B and D).

Damage below the red nucleus (eg, pons) or to the nucleus itself (eg, midbrain) often causes decerebrate (extensor) posturing due to loss of descending excitation to the upper limb flexors (via the rubrospinal tract) and extensor predominance (due to unopposed vestibulospinal tract output).

With respect to skeletal muscle, the afterload refers to the force against which the muscle fiber must contract. This is analogous to the afterload caused by diastolic pressure in cardiac ventricular contraction. When a skeletal muscle fiber contracts, its maximum velocity of shortening is indirectly proportionate to the afterload attached to the fiber

Decrease afterload -> increase velocity

Depletion of donor CD3+ T cells prior to HSC transplant is performed primarily to decrease GVHD risk, but it also has the following undesirable effects on the host

Decreased graft versus leukemia effect, increasing the risk of leukemia relapse (Choice D). Depletion of memory T cells, which increases the risk of viral (eg, cytomegalovirus, human herpes virus 6) infections (Choice A). This also increases the risk of posttransplant lymphoproliferative disease due to the reduction in donor T cells that normally attack proliferating B cells infected with Epstein-Barr virus Allogeneic stem cell transplantation is used in the treatment of leukemia to provide a new source of hematopoietic stem cells (HSCs) after

What causes hypercholesterolemia in hypothyroidism?

Decreased synthesis of LDL receptors

Myasthenia gravis (MG) is caused by autoantibodies against postsynaptic nicotinic acetylcholine receptors, leading to fewer functional receptors and fatigable muscle weakness. Nondepolarizing neuromuscular blocking agents (eg, vecuronium) are competitive antagonists of nicotinic receptors; due to the depletion of receptors, patients with MG are extremely sensitive to these agents

Depolarizing agents (eg, succinylcholine) are competitive agonists of nicotinic acetylcholine receptors; persistent depolarization leads to desensitization of the motor end plate. Due to the low number of functioning receptors, patients with MG are often resistant to these agents; extremely large doses are required to induce muscle relaxation Venous insufficiency is a multifactorial process; contributing factors include obstruction of venous return (eg, pregnancy, obesity), increased orthostatic pressure (eg, prolonged standing), decreased muscular compression (eg, sedentary lifestyle), and damaged venous valves (eg, due to deep vein thrombosis). Over time, patients with venous insufficiency can develop extravasation of fluid, plasma proteins, and red cells, leading to chronic inflammation and stasis dermatitis (SD). SD is characterized by erythema, induration, fibrosis, and deposition of hemosiderin (from breakdown of extravasated RBCs) manifesting as reddish-brown discoloration, as in this patient. Symptoms are typically bilateral and usually worse at or above the ankles. Complications of SD include poor wound healing and ulceration.

Lower extremity differential clubbing and cyanosis without pulse discrepancy is caused by a large patent ductus arteriosus complicated by Eisenmenger syndrome (reversal of shunt flow from left-to-right to right-to-left). In contrast, intracardiac right-to-left shunting in patients with large septal defects and Tetralogy of Fallot results in whole-body cyanosis.

Differential clubbing and cyanosis can also occur with uncorrected transposition of the great arteries when there is extracardiac mixing through a PDA. However, cyanosis is present in the upper extremities; the lower extremities are better oxygenated due to flow from the PDA

The most common cause of an abnormal AFP level is inaccurate pregnancy dating (ie, dating error). In patients with irregular menses, dating by a last menstrual period can underestimate the true gestational age. Therefore, these patients require a fetal ultrasound, which can accurately determine gestational age and evaluate for other common causes of elevated MSAFP levels, which include multiple gestation (eg, twin pregnancy), open neural tube defects, and abdominal wall defects.

Down syndrome (trisomy 21) and Edwards syndrome (trisomy 18) are associated with low AFP levels.

With urinary catheters what is most significant risk factor for UTI

Duration of catheterization is the most significant risk factor for UTI. Preventive measures include avoiding unnecessary catheterization, using sterile technique when inserting the catheter, and removing the catheter promptly when no longer needed. Urinary tract infections (UTIs) are common in hospitalized patients with indwelling urinary catheters.

Cleft palate

During the seventh-eighth week of embryonic development, the maxillary prominences give rise to palatine shelves. The thin sheets of tissue that comprise the palatine shelves grow medially and fuse into the secondary palate. Fusion of the secondary palate with the posterior aspect of the primary palate forms the complete palate. Failure of any of these processes can lead to cleft palate.

Mutation associated with glioblastoma multiform

EGFR (also PTEN)

the immunodiffusion test used to identify toxin-producing strains of Corynebacterium diphtheriae.

ELEK Test

The rubber-like properties of elastin are due to high content of nonpolar (hydrophobic) amino acids and extensive cross-linking between elastin monomers facilitated by lysyl oxidase.

Elastin also contains proline and lysine residues; however, in contrast to those found in collagen, few of these amino acids are hydroxylated

Pulmonary edema in left heart failure leads to what finding in the lung Hemosiderin-laden macrophages

Elevated pulmonary venous pressure leads to transudation of fluid across the alveolar-capillary membrane (pulmonary edema) and can cause breaks in the endothelium with extravasation of red blood cells into the alveoli and lung parenchyma. As alveolar macrophages engulf and degrade the extravasated red blood cells, the released iron accumulates as intracellular hemosiderin.

Vasospastic angina is the result of intermittent coronary vasospasm caused by endothelial dysfunction and autonomic imbalance. Although acetylcholine normally stimulates vasodilation, it triggers coronary vasospasm in affected patients due to a deficiency of vasodilatory endothelial nitric oxide.

Ergot alkaloids activate 5-HT2 serotonergic receptors to cause vasoconstriction. Normally, the vasoconstriction is somewhat offset by endothelial release of vasodilatory prostaglandins, but this response is lacking in affected patients due to endothelial dysfunction.

Etanercept s. Other TNFa drugs

Etanercept is a fusion protein with domains derived from the Fc portion of IgG1 and TNF receptor 2. It functions as a decoy receptor for TNF-α. Other TNF-α inhibitors are anti-TNF monoclonal antibodies (eg, infliximab, adalimumab)

Malignant transformation, in contrast, is monoclonal in that it results from the unchecked proliferation of a single genetically unique cell from only one cell line.

Evaluation for monoclonality of the lymphocyte population is important when lymphoma is suspected. The clonality of a T-cell population is assessed by molecular methods, such as PCR, that examine the rearrangement of T-cell receptor (TCR) genes. If a single allele for the V region of the T-cell receptor predominates in a lymphocytic population, monoclonal proliferation is suspected. The same principle applies when assessing B-cell clonality. Monoclonal rearrangement of the genes for immunoglobulin variable regions is suggestive of a B-cell lymphoma

vitamin D-resistant rickets has an X-linked dominant inheritance pattern.

Examples: fragile X syndrome, Alport syndrome, hypophosphatemic rickets (also called X-linked hypophosphatemia)—phosphate wasting at proximal tubulerickets-like presentatio

Chronic kidney disease (CKD) results in impaired conversion of 25-hydroxyvitamin D to 1,25-dihydroxyvitamin D due to the following factors:

FGF23 levels increase early in CKD, causing direct inhibition of 1-alpha-hydroxylase Reduced glomerular filtration limits the delivery of 25-hydroxyvitamin D to proximal tubule cells Reduced functional renal mass limits production of 1-alpha-hydroxylase This leads to a compensatory rise in parathyroid hormone (secondary hyperparathyroidism) that can present with weakness, muscle and joint pain, defective bone mineralization, and increased fracture risk

Gastric varices are dilated submucosal veins that can cause life-threatening bleeding in the upper gastrointestinal tract. The varices are commonly due to portal hypertension, which can be a complication of cirrhosis. Gastric varices can also be seen with splenic vein thrombosis due to chronic pancreatitis, pancreatic cancer, and abdominal tumors. The splenic vein runs along the posterior surface of the pancreas and can develop a blood clot from pancreatic inflammation. The short gastric veins drain the fundus of the stomach into the splenic vein. Splenic vein thrombosis can increase pressure in the short gastric veins and cause gastric varices only in the fundus. The rest of the stomach and esophagus are usually not affected

Failure of the urachus to obliterate before birth leads to several abnormalities: Patent urachus, urachal sinus, urachal cyst Complete failure of obliteration of the urachus results in a patent urachus that connects the umbilicus and bladder. Patients present with straw-colored urine discharge from the umbilicus, which is exacerbated by crying, straining, or prone position. Local skin irritation can cause erythema. Failure to close the distal part of the urachus (adjacent to the umbilicus) results in a urachal sinus. This presents with periumbilical tenderness and purulent umbilical discharge due to persistent and recurrent infection. Failure of the central portion of the urachus to obliterate leads to a urachal cyst.

Norovirus is the most common cause of viral gastroenteritis. Outbreaks in crowded settings (eg, schools, cruise ships, nursing homes) are common, and symptoms include vomiting and watery diarrhea.

Focal nodular hyperplasia is a benign liver tumor marked by a central stellate scar containing an abnormally large artery. It usually arises in young women and most cases are asymptomatic and found incidentally Hepatic adenoma is a benign liver lesion that is strongly associated with oral contraceptive use and may undergo malignant transformation or rupture. Although it can also be discovered incidentally, a hepatic adenoma will usually appear as an irregular, tan-colored group of nodules composed of large plates of adenoma cells with an absence of normal hepatic architecture (no portal structures or bile ducts)

GABA is the primary inhibitory neurotransmitter in the CNS and acts on two different receptors:

GABA A receptor mediates a fast response to GABA because it is an ion-gated chloride channel. Activation of this receptor leads to rapid muscle relaxation and sedation. Its activity is enhanced by benzodiazepines, which can be used to treat spasticity. GABA B receptor mediates a slower response to GABA because it is mediated by a G protein-coupled receptor that opens potassium channels. It is agonized by baclofen, which can also be used to treat spasticity.

Field cancerization describes a large area of cells within a field (eg, upper aerodigestive tract) that are "primed" to develop cancer because of widespread exposure to mutagens (eg, tobacco). Mutations that affect many similar cells can increase the likelihood of developing multiple primary malignancies, analogous to germline mutations in tumor suppressor genes (eg, BRCA1 or BRCA2 in breast and ovarian cancer). Field cancerization likely underlies the development of many common cancers, including:

HNSCC due to mucosal exposure to tobacco carcinogens Basal cell carcinoma of the skin due to skin exposure to ultraviolet light Colorectal carcinoma due to epithelial exposure to dietary carcinogens Even with adequate treatment of a malignancy, the areas affected by field cancerization remain at higher risk of developing multiple primary malignancies In the event of a nuclear accident, potassium iodide is given prophylactically to protect the thyroid from excessive accumulation of radioactive 131I.

Exogenous T3 supplementation rapidly suppresses TSH levels by increasing negative feedback, which in turn decreases T4 secretion from the thyroid gland. Furthermore, rT3 also decreases because less T4 is available for conversion (T3 cannot be converted into rT3).

However, synthetic T3 (liothyronine) is not recommended for the routine treatment of hypothyroidism, as it has a short half-life and patients can experience wide fluctuations in plasma T3 levels. T4 (levothyroxine) supplementation provides a more physiologic effect and is preferred.

In IPF, type 2 pneumocytes undergo reactive hyperplasia but fail to differentiate into type 1 cells because of dysfunctional cell fate pathways (eg, Wnt/transforming growth factor-beta) and abnormalities of the underlying basement membrane

In IPF, repetitive microinjury to the alveolar epithelium (eg, smoking, acid reflux) is focally repaired by fibroblast proliferation/collagen deposition instead of normal restoration via type 2 pneumocyte differentiation.

The most effective method of preventing anthracycline-induced cardiomyopathy is the administration of dexrazoxane, a chelating agent thought to both block the formation of iron-associated free radicals and inhibit the formation of anthracycline-topoisomerase II complexes in healthy cardiomyocytes

In a patient with achalasia, injection of botulinum toxin into the LES causes cholinergic blockade, leading to sphincter relaxation and symptomatic improvement.

Acute cholecystitis is most often caused by gallstones obstructing the cystic duct. The diagnosis can be made by identifying signs of gallbladder inflammation (eg, wall thickening, pericholecystic fluid) on ultrasonography. When ultrasound is inconclusive, nuclear medicine hepatobiliary scanning (ie, cholescintigraphy) can be used to assess cystic duct patency and make the diagnosis.

In acute or chronic cholecystitis, the radiotracer will be taken up by the liver with progressive excretion into the common bile duct and proximal small bowel, but the gallbladder will not be visualized due to the obstruction

Photoaging is a product of excess exposure to ultraviolet A wavelengths and is characterized by epidermal atrophy with flattening of rete ridges. In addition, there is decreased collagen fibril production and increased degradation of collagen and elastin in the dermis

In addition, there is increased crosslinking of collagen (Choice A), with deposition of collagen breakdown products. The atrophic dermis and increased collagen crosslinking, along with desiccation of the stratum corneum, produce the characteristic wrinkling of photoaged skin.

Classically, the stimulation of acid secretion within the stomach is separated into three phases: cephalic, gastric, and intestinal. The cephalic phase is mediated primarily by cholinergic and vagal mechanisms, and is triggered by the thought, sight, smell, and taste of food. The gastric phase is mediated by the presence of gastrin (which stimulates histamine secretion and therefore, indirectly, acid secretion), and is triggered by the chemical stimulus of food and distension of the stomach. The intestinal phase is initiated when protein-containing food enters the duodenum, but this phase plays only a minor role in stimulating gastric acid secretion.

In fact, intestinal influences are effective in down-regulating gastric acid secretion after a meal.

Wound healing normally progress through the inflammatory, proliferative, and remodeling phases. In the days after the initial injury, neutrophils and other immune cells are recruited to the wound and produce an inflammatory response that helps prevent bacterial overgrowth in the nutrient-rich environment of a healing wound. However, this inflammation also impairs formation of granulation tissue that is needed for normal wound healing. As healing progresses, release of growth factors and anti-inflammatory cytokines (eg, IL-10) by macrophages and regulatory T cells suppresses the inflammatory response, facilitating fibroblast proliferation and reepithelialization of the wound.

In patients with diabetes mellitus, constitutively elevated blood glucose increases inflammation by stimulating the release of proinflammatory cytokines and reactive oxygen species from neutrophils (Choice E). Elevated glucose also leads to a marked decrease in IL-10 production that contributes to the increased susceptibility for chronic, nonhealing wounds and ulcers in patients with uncontrolled diabetes.

Removal of excess cholesterol from the body occurs via 2 mechanisms: excretion of free cholesterol into bile and conversion of cholesterol into bile acids.

In the liver, free cholesterol is converted into cholic and chenodeoxycholic acids through a series of chemical reactions beginning with cholesterol 7α-hydroxylase (rate-limiting step in bile acid synthesis). These bile acids are then conjugated to either glycine or taurine (improving solubility and emulsifying ability) to create the bile salts that are actively secreted into the bile canaliculi. As water-insoluble cholesterol is secreted in bile, it is rendered soluble in small amounts by the detergent action of these amphipathic (eg, hydrophobic and hydrophilic) bile salts and phosphatidylcholine (a phospholipid). When there is more cholesterol than can be made soluble, it precipitates into crystals that eventually grow and merge to form gallstones. Gallbladder hypomotility further promotes cholesterol nucleation and gallstone formation.

In anemia what is change in HR, SV, CO, pulse pressure

Increase HR, Increase SV, Increase CO, widened pulse pressure Anemia creates increased demand for cardiac output (CO), which is met by an increase in both heart rate (HR) and stroke volume (SV; CO = HR x SV). Pulse pressure characteristically widens due to the increased CO and a simultaneous decrease in systemic vascular resistance (SVR).

The tissue tropism of viruses is primarily mediated by viral surface glycoproteins that bind to specific host cell receptors. Mutations to viral surface glycoproteins can alter tissue tropism and cause noninfective viruses to become infectious

Influenza strains that are unable to infect humans (eg, avian/swine influenza) encode for hemagglutinin that cannot bind to human cells. Interaction with the host cell is dependent upon a viral surface glycoprotein called hemagglutinin, which binds to the sialic acid receptor on human respiratory epithelial cells.

Patients with ALS most often die of respiratory failure, which is caused by multiple factors:

Inspiratory muscle (eg, diaphragm) atrophy and weakness, which leads to poor inspiratory strength. This is evident on pulmonary function testing as decreased vital capacity (the maximum volume displaced from the lung), which is followed as a marker of disease progression in patients with ALS. Expiratory muscle weakness, which leads to an ineffective cough and poor airway clearance. Bulbar muscle weakness, which leads to dysphagia and chronic aspiration. Treatment of ALS often focuses on improving respiratory function, which both prolongs survival and improves quality of life.

Up to 80% of patients with SS develop pulmonary complications, with the incidence depending on the SS subtype:

Interstitial lung disease is the most common pulmonary complication of diffuse cutaneous SS (ie, skin involvement extends proximal to the wrist and may include the trunk). Pulmonary hypertension is the most common pulmonary complication of limited cutaneous SS (ie, skin involvement is mainly in the hands and/or face), which is often associated with CREST syndrome

Sensory innervation of the remainder of the parietal pleura is accomplished by intercostal nerves and is typically felt closer to the source of the pain (Choice B).

Irritation of the parietal pleura will cause sharp pain, which is worse on inspiration. Pain arising from the mediastinal or diaphragmatic pleura will be carried by the phrenic nerve and referred to the C3-C5 distribution.

Meckel's diverticulum -omphalomesenteric (vitelline) duct

It also often contains ectopic mucosa−gastric epithelium being the most common (pancreatic tissue is the next most common). This ectopic gastric tissue secretes gastric acid that can cause ulceration of adjacent mucosa and lower GI bleeding (melena/hematochezia).

In diabetic ketoacidosis, the level of 3-hydroxybutyrate increases approximately 5 times the increase in acetoacetate.

Ketoacids, which include acetoacetate and 3-hydroxybutyrate, are normally produced in roughly equal amounts. However, in pathologic conditions such as poorly controlled diabetes and alcoholism, in which the concentration of NADH is high, the ratio shifts and 3-hydroxybutyrate predominates. Therefore, the ratio of acetoacetate to 3-hydroxybutyrate decreases, while the net amount of ketoacids increases. In this situation the physician should specifically request a lab measurement of 3-hydroxybutyrate to have a more accurate measurement of the extent of the initial level on admission and to more accurately follow the resolution during treatment. This is because bedside dip stick tests do not measure 3-hydroxybutyrate. In diabetic ketosis, NADH increases as a result of increased fatty acid beta-oxidation, which supplies the acetyl CoA for ketogenesisincreases of right atrial pressure above left atrial pressure can produce a right-to-left shunt, leading to paradoxical embolism of venous clots into the arterial circulation

Chlorpromazine MOA

Low potency antipsychotic

Thioridazine MOA

Low potency antipsychotic

myelodysplastic syndrome (MDS). MDS is a clonal hematologic malignancy associated with the development of driver mutations due to advanced age, previous chemo-/radiotherapy, or exposure to environmental toxins.

MDS is marked by the following: * ≥1 cytopenias: normal hematopoiesis is impaired due to neoplastic cell replication in the bone marrow; therefore, patients usually present with symptoms of ≥1 cytopenia such as fatigue/dyspnea on exertion (anemia), bleeding/bruising (thrombocytopenia), or infections (leukopenia). Because erythrocyte production is impaired, reticulocyte count will be low despite significant anemia. However, significant extramedullary hematopoiesis does not occur, so hepatosplenomegaly is rare. * Dysplasia of erythrocytes and neutrophils: peripheral blood smear usually shows normocytic or macrocytic erythrocytes with a variety of abnormalities (eg, oval macrocytes). Neutrophils are typically hypolobulated and hypogranular. * Bone marrow biopsy typically reveals a hypercellular marrow with dysplasia of ≥1 cell lines. Myeloblasts are increased (but <20% of total cells), and granulocytes show evidence of impaired maturation such as abnormalities in size, granulation, or lobulation. Erythrocytes also show signs of abnormal development, including large size and nuclear lobation/budding.

Acne vulgaris is characterized by inflammation of pilosebaceous follicles. Contributing factors include hyperkeratinization and obstruction of follicles; sebaceous gland enlargement with increased sebum secretion; colonization of the gland by Propionibacterium (Cutibacterium) acnes, which metabolizes lipids in sebum and produces proinflammatory factors; and follicular and perifollicular inflammation with follicular rupture

MM cells are particularly susceptible to proteasome inhibition because they generate extremely high quantities of secretory proteins (eg, monoclonal immunoglobulins), some of which misfold and aggregate within the endoplasmic reticulum. Increased stress on the endoplasmic reticulum triggers an unfolded protein response that eventually leads to activation of caspases and cellular apoptosis

Renal effects of preeclampsia include oliguria with increased specific gravity, proteinuria, and serum creatinine levels.

Microscopic hematuria (ie, red blood cells on urinalysis) is typically associated with urinary tract disease, such as infection, kidney or bladder cancer, and renal calculus. Hematuria is not associated with hypertension in pregnancy

clusters of firm, shiny, round papules that are each 2-6 mm in diameter with central indentation

Microscopy of the lesions shows intracytoplasmic eosinophilic inclusion bodies (molluscum bodies) composed of poxvirus within keratinocytes in the epidermis.

Healthy worker bias occurs in cohort studies of occupational exposures in an employed population when the comparison group is the general population. Compared to the employed population (ie, healthy enough to work), the general population can include unhealthy subjects (ie, not healthy enough to work); therefore, the mortality rate in the general population may be higher than in an employed population, and comparing mortality rates between these 2 cohorts is biased.

Misclassification bias results from an incorrect categorization of subjects regarding their exposure status, outcome status, or both. Nondifferential misclassification reflects equal misclassification rates between groups, making them look more similar than they are and pushing the association toward the null value (eg, odds ratio = 1) (ie, bias toward null hypothesis) Differential misclassification reflects unequal misclassification rates between groups and can push the association toward or away from the null value

List Encephalitis Viruses

Mosquito-borne Arboviruses Flaviviruses (birds) • St. Louis • West Nile Togaviruses (birds) • Eastern Equine • Western Equine Bunyaviruses (rodents) • California

subclavian steal syndrome, which typically occurs due to hemodynamically significant stenosis of the subclavian artery proximal to the origin of the vertebral artery. Subclavian stenosis is typically caused by atherosclerosis, although less common etiologies include Takayasu arteritis and complications from heart surgery (eg, aortic coarctation repair). The lowered distal subclavian arterial pressure leads to reversal in blood flow ("steal") from the contralateral vertebral artery to the ipsilateral vertebral artery, away from the brainstem.

Most patients with subclavian artery stenosis are asymptomatic. When symptoms occur, they are typically related to arm ischemia in the affected extremity (eg, exercise-induced fatigue, pain, paresthesias) or vertebrobasilar insufficiency (eg, dizziness, vertigo, drop attacks). Physical examination can show a significant difference (>15 mm Hg) in brachial systolic blood pressure between the affected arm and normal arm. Doppler ultrasound of the cerebrovascular and upper extremity arterial circulation establishes the diagnosis in most patients. innominate (brachiocephalic) artery stenosis or occlusion can cause subclavian steal syndrome, patients with significant occlusion would develop retrograde flow through the right vertebral artery rather than the left

Systemic chemotherapy targets rapidly dividing cancer cells but often incidentally damages other rapidly dividing cells in the gastrointestinal tract and bone marrow, which dramatically increases the risk of infection due to:

Mucositis - Damage to rapidly dividing epithelial cells of the digestive tract creates breaches in the mucosal barrier that allow endogenous bacterial flora (commensal organisms) to penetrate and enter the bloodstream. Gram-negative enteric bacilli, such as Pseudomonas aeruginosa, and gram-positive skin organisms, such as Staphylococcus epidermidis and S aureus, are most likely to invade. Neutropenia - Damage to rapidly dividing granulopoietic cells reduces circulating neutrophil count. Because neutrophils serve as front-line defense against bacterial and fungal pathogens, commensal pathogens that penetrate the mucosal/cutaneous barrier cannot be contained and subsequently spread to the bloodstream and deeper tissues. Neutropenic patients have weak inflammatory responses and fever is often the only sign of infection (neutropenic fever). To reduce the risk of serious infection, patients who receive systemic chemotherapy are often empirically treated with prophylactic antibiotics (eg, ciprofloxacin) to prevent commensal organisms from invading and with granulocyte-stimulating agents (eg, filgrastim) to increase neutrophil count

Symptoms of neonatal abstinence syndrome

Neurologic: Irritability, hypertonia, jittery movements, seizures (rare) Gastrointestinal: Diarrhea, vomiting, feeding intolerance Autonomic: Sweating, sneezing, pupillary dilation

Capsaicin cream is used to treat _ and how

Neuropathic pain/Post herpetic neuralgia. Decreases substance p (Loss of membrane potential in nociceptive fibers) Capsaicin is an irritant found in the chili pepper family. It causes excessive activation of TRPV1 (a transmembrane cation channel), causing a buildup of intracellular calcium that results in long-lasting dysfunction of nociceptive nerve fibers (defunctionalization). In addition, capsaicin causes release and subsequent depletion of substance P, a polypeptide neurotransmitter involved in transmission of pain signals. On initial application, topical capsaicin causes burning, stinging, and erythema, but persistent exposure leads to a moderate reduction in pain over time.

With both acute and chronic HP, BAL usually shows high relative lymphocyte count (eg, >20%, often >50%), which helps support the diagnosis. Other causes of high relative lymphocyte count in BAL include sarcoidosis, lymphoma, and chronic fungal or mycobacterial infection.

Normally, the leukocytes in alveolar fluid consist of approximately 85% alveolar macrophages, 10% lymphocytes, and a small percentage of neutrophils and eosinophils

An unwanted effect of the hyperventilation is excessive expiration of CO2, leading to respiratory alkalosis and increased blood pH. The hypoxemia and alkalemia can cause altitude sickness, which presents with headache, fatigue, lightheadedness, nausea, and insomnia.

Over time, the body makes physiologic adjustments to better tolerate high altitude: In response to respiratory alkalosis, the kidneys decrease HCO3- reabsorption and H+ secretion to create a compensatory metabolic acidosis (a process that begins within hours and requires several days to complete). Alkalosis also causes a left shift in the hemoglobin dissociation curve, which initially impairs tissue oxygen delivery; however, the curve is shifted back to the right by increased production of 2,3-biphosphoglycerate in red blood cells, facilitating O2 unloading. Chronic hypoxemia triggers increased erythropoietin secretion by the kidneys with a resulting increase in red blood cell production (a process that requires several weeks). On initial exposure to high altitude, the administration of a carbonic anhydrase inhibitor (eg, acetazolamide) can accelerate the decrease in HCO3- reabsorption by the kidneys to help relieve the alkalemia and treat altitude sickness.

Chronic Chagas cardiomyopathy is the most common complication of Chagas disease. It is thought to be caused by chronic, low-grade, parasite-mediated myocarditis, which leads to the progressive destruction of cardiac fibers and subsequent cardiac fibrosis. Common complications include biventricular heart failure, cardiac arrhythmias (particularly ventricular arrhythmias), and ventricular aneurysm with intracardiac thrombus (leads to thromboembolic disease/stroke).

Patients in rural areas are at greatest risk when their houses harbor the vector (Triatome bug) in adobe walls or thatched roofs.

Stress urinary incontinence (intermittent, involuntary leakage of urine) is caused by weakened pelvic floor muscle support that often occurs due to chronically increased intraabdominal pressure (eg, obesity, chronic cough, prior pregnancies).

Patients with severe pelvic floor laxity can also develop herniation of the bladder into the vagina (ie, cystocele), which further worsens SUI symptoms.

Chronic pelvic pain in a multiparous woman that increases with standing and leads to painful intercourse is consistent with pelvic congestion syndrome.

Pelvic congestion is due to dilated veins or varicosities within the pelvis, from incompetent veins, and/or proximal obstruction. Imaging would show dilated veins within the pelvis.

Patients infected with HIV-2 have much lower levels of viremia than those infected with HIV-1. Because viremia is directly correlated with transmission risk, rate of CD4 count decline, and progression to AIDS, individuals with HIV-2 are usually less infectious and have longer asymptomatic periods than patients with HIV-1. The diagnosis of HIV-2 is often suspected when HIV test results are incongruent; most patients will have the following laboratory results:

Positive fourth-generation HIV testing (HIV antigen/antibody test) - This test evaluates for the presence of p24 antigen and HIV-1 and HIV-2 antibodies but does not usually differentiate between HIV-1 and HIV-2. Indeterminate or negative HIV-1 confirmatory testing - HIV-1 Western blot evaluates for proteins in HIV-1 and is usually indeterminate in the setting of HIV-2 infection. Plasma HIV-1 RNA testing will be negative

The absolute contraindications to the use of OCPs are:

Prior history of thromboembolic event or stroke History of an estrogen-dependent tumor Women over age 35 years who smoke heavily Hypertriglyceridemia Decompensated or active liver disease (would impair steroid metabolism) Pregnancy

anesthesia must be induced. Three medications are commonly used because of their rapid onset of action and short duration of effect.

Propofol: A highly lipophilic GABA agonist that may be used for long-term sedation. Disadvantages include vasodilation, which can result in hypotension and an increase in serum triglycerides and lipase. Etomidate: A GABA agonist that has the advantage of being the most hemodynamically neutral. It does not cause changes in heart rate, blood pressure, or cardiac output. However, it inhibits cortisol synthesis, which can lead to (reversible) adrenocortical suppression. Because of this, it is often avoided in patients with septic shock, and it should not be used as maintenance of sedation after induction. Ketamine: An N-methyl-D-aspartate (NMDA) antagonist that is similar to PCP; it preserves the respiratory drive during induction of anesthesia. In contrast to propofol and etomidate, it also provides an analgesic effect. Ketamine stimulates the release of catecholamines (ie, sympathomimetic), which can cause bronchodilation but also increase the heart rate, myocardial contractility, and cerebral blood flow, potentially putting the patient at risk for cardiovascular events or increased intracranial pressure.

The net filtration pressure is a result of pressure gradients formed by Starling forces and is calculated by subtracting the oncotic pressure gradient from the hydrostatic pressure gradient. Angiotensin II preferentially constricts the efferent arteriole, resulting in an increased hydrostatic pressure gradient and an increased net filtration pressure.

Prostaglandins (eg, prostaglandin E2) are responsible for dilation of the afferent arteriole. Inhibition of prostaglandin synthesis, as seen with nonsteroidal anti-inflammatory drugs, results in constriction of the afferent arteriole, leading to reduced hydrostatic pressure and a lower net filtration pressure. Alpha-1 receptors are located mainly in the afferent arteriole; alpha agonists (eg, epinephrine, norepinephrine) result in constriction of the afferent arteriole, which reduces hydrostatic pressure and leads to lower net filtration pressures.

Alcohol-based disinfectants, which are composed of ethyl or isopropyl alcohol (at concentrations of 60%-90%) in water, are often used for hand hygiene. These solutions kill vegetative bacteria (but not spores), fungus, and enveloped viruses (eg, influenza, HIV, herpes) by dissolving their lipid bilayer membranes and subsequently denaturing their proteins. Nonenveloped viruses such as adenovirus, rhinovirus, enterovirus, and poliovirus are less susceptible to some alcohol-based disinfectants (particularly isopropyl-based solutions) because they do not have a lipid bilayer envelope

Proteins (and spores) are sterilized by moist heat (eg, autoclave), which results in protein denaturation. Although proteins are somewhat susceptible to alcohol-based disinfectants, high concentrations must be used and efficacy is variable. Alcohol has the strongest effect on lipophilic membranes.

snRNA is transcribed by

RNA polymerase II and associates with specific proteins to form snRNPs.

ARR = control rate - treatment rate

RRR = ARR/control rate RR = treatment rate/control rate NNT = 1/ARR

What innervates the supinator muscle

Radial nerve

Melatonin receptor agonist; binds MT1 and MT2 in suprachiasmatic nucleus.

Ramelteon

Color variegation is a common feature of melanoma; the different colors represent different areas of activity within the tumor:

Red areas are due to vessel ectasia (dilation) and local inflammation (Choices A and E). Brown or black, flared areas along the border are due to advancing, neoplastic melanocytes (Choice C). White and gray areas appear when cytotoxic T lymphocytes recognize tumor antigens (eg, melan-A) and induce apoptosis, leading to malignant melanocyte regression (cleared patches). Melanocyte regression is also a hallmark of treatment response to melanoma immunotherapies, such as the programmed cell death receptor-1 (PD-1) inhibitor pembrolizumab Preeclampsia likely occurs due to abnormal placental development that results in high-resistance, low-perfusion vessels and subsequent placental ischemia. Placental ischemia triggers widespread endothelial dysfunction, which causes dysregulated vascular tone (eg, vasospasm, vasoconstriction), increased vascular permeability, and decreased end-organ perfusion. Therefore, a classic presentation of preeclampsia is hypertension with headache and visual changes: Headaches are typically severe and throbbing in nature. They occur due to dysregulated cerebral blood flow, brain ischemia, and/or capillary leakage resulting in cerebral edema. Visual changes include blurry vision, photopsia (seeing "sparks"), and scotomata (partial vision loss [blind spots]) and are due to retinal artery vasospasm and optic nerve ischemia. Eye movement and the pupillary light reflex typically remain intact. Symptoms usually resolve after delivery, which is curative for preeclampsia. However, in rare cases, patients with severe retinal ischemia or retinal detachment may have permanent vision loss.

How does malnourishment cause hypophosphatemia

Redistribution of phosphate from the serum into muscle and hepatic cells Malnourishment (eg, due to chronic alcohol use disorder) results in the depletion of phosphate, although serum levels may remain normal due to transcellular shifts. Reintroduction of carbohydrates (ie, dextrose-containing intravenous fluids) increases insulin secretion, which stimulates the redistribution of phosphate from the serum into muscle and hepatic cells for use during glycolysis (eg, formation of ATP, 2-3 diphosphoglycerate). This leads to profound hypophosphatemia; lack of adequate intracellular phosphate can result in failure of cellular energy metabolism, producing the clinical features of refeeding syndrome (eg, muscular weakness, arrhythmias, congestive heart failure).

In pregnancy, how can Progesterone predispose to gallstones?

Reduces bile acid secretion Slows gallbladder emptying

Left-sided varicoceles are relatively common in healthy pubertal men as the aorta and superior mesenteric artery can compress the left renal vein ("nutcracker effect"), resulting in increased intravascular pressure in the left gonadal vein with retrograde blood flow and varicocele formation. However, a new-onset left varicocele associated with ipsilateral flank pain and hematuria should raise suspicion for

Renal vein thrombosis (RVT) causing impaired left gonadal venous drainage.

Breast cancer involvement of the suspensory ligaments produces which physical exam finding?

Skin retractions (ie, Cooper ligaments). Malignant infiltration of these ligaments causes fibrosis and shortening, leading to traction on the skin with distortion in breast contour. Dermatophyte infections (eg, tinea corporis, tinea pedis, tinea cruris) infect keratinized matter in the stratum corneum of the superficial epidermis but do not invade the dermis and subcutaneous tissues. mildly pruritic, polycyclic rash with a raised, scaly border and central clearing, has tinea corporis.

Stimulation of what nerve using an implantable nerve stimulator can improve obstructive sleep apnea

Stimulation of the hypoglossal nerve using an implantable nerve stimulator causes the tongue to move forward slightly, increasing the anteroposterior diameter of the airway. Studies with these devices have shown a reduction in the number of obstructive events during sleep.

Purulent pericarditis is usually caused by hematogenous dissemination from distant infection or direct extension from an adjacent infection or chest wall trauma. Although a variety of organisms cause purulent pericarditis, Staphylococcus aureus is the most common pathogen, particularly in the setting of a portal from the skin to the bloodstream (eg, catheter) or pericardium (eg, recent chest surgery, penetrating injury).

Streptococcus pneumoniae is the most common organism in patients with adjacent pneumonia. Candida albicans Streptococcus pneumoniae (adjacent pneumonia) Candida albicans (TPN, severe immunosuppression)

Describe phases of Succinylcholine action

Succinylcholine is a fast-acting, depolarizing neuromuscular blocking agent used for rapid-sequence intubation that causes equal reduction of all 4 twitches during train-of-four stimulation (phase I blockade). Prolonged administration of succinylcholine or use in patients with abnormal plasma cholinesterase activity causes transition to a phase II (nondepolarizing) block, seen as a progressive reduction in each of the 4 twitches. Train-of-four (TOF) stimulation is used during anesthesia to assess the degree of paralysis induced by neuromuscular junction (NMJ)-blocking agents. A peripheral nerve is stimulated 4 times in quick succession and the muscular response is recorded. The height of each bar represents the strength of each twitch; higher bars indicate the activation of increasing numbers of individual muscle fibers (myocytes).

Ethylene glycol is a toxic alcohol that is metabolized to glycolate, which is cytotoxic to the renal tubules and causes acute tubular necrosis. Glycolate is further metabolized to oxalate, which contributes to oliguric renal failure by precipitating in the kidneys and causing tubular obstruction. Urine microscopy will show envelope- or dumbbell-shaped calcium oxalate crystals that are positively birefringent on polarization.

Sudden-onset back pain, hematuria, and oliguria associated with high anion gap metabolic acidosis and osmolar gap suggest ethylene glycol poisoning. Urine microscopy will show envelope- or dumbbell-shaped calcium oxalate crystals.

Chronic glucocorticoid use is characterized by low CRH, ACTH, and cortisol levels that cannot rise in response to stressful situations (eg, infections, surgery)

Suppression of the hypothalamus-pituitary-adrenal axis by glucocorticoid therapy is the most common cause of adrenal insufficiency. In these patients, adrenal crisis can be precipitated by stressful situations (eg, infections, surgery) if the glucocorticoid dose is not increased appropriately.

T-cell senescence is marked by minimal response to stimuli, it occurs due to the shortening of telomeres after several rounds of T-cell replication

T cells that bind an antigen on an APC but are not costimulated undergo anergy, a form of peripheral immune tolerance marked by a lack of T-cell response to cytokine, ligand, or antigen stimulation (T-cell inactivation). Because CTLA4 binds to CD80/86 with greater affinity than CD28, administration of exogenous CTLA4 (eg, abatacept) reduces the availability of ligands necessary for T cell costimulation, which dramatically increases T-cell anergy in areas of active inflammation.

Tuberculous (TB) meningitis is characterized by formation of a thick, gelatinous exudate in the base of the brain; cerebral vasculitis; and hydrocephalus. It frequently presents with subacute, slowly progressive nausea, vomiting, fever, cranial nerve deficits, and strokes.

TB meningitis arises when circulating bacilli lodge in the subependymal or subpial space, forming a tubercle that subsequently ruptures into the subarachnoid space. Common findings include the following:

The thoracic outlet refers to the region above the first rib and behind the clavicle that is bordered by the cervical vertebral bones and the sternum. Compression of the brachial plexus as it passes through the thoracic outlet can cause upper extremity numbness, tingling, and weakness (most often in an ulnar distribution due to compression of the lower trunk). Involvement of the subclavian vein and artery can also occur and lead to upper extremity swelling and exertional arm pain.

TOS most commonly occurs due to compression of the brachial plexus within the scalene triangle, which is formed by the anterior and middle scalene muscles and the first rib.

Because of their large molecular size, mAbs cannot be administered orally and must be given via intravenous or subcutaneous/intramuscular routes. Unlike most other drugs, mAbs are not eliminated by hepatic or renal clearance, but are instead removed from the body in 2 primary ways:

Target-mediated drug clearance: mAbs directed against cell surface antigens undergo internalization (receptor-mediated endocytosis) upon binding to their targets, removing them from the circulation Nonspecific clearance: Immunoglobulins are constitutively taken up by reticuloendothelial macrophages (via binding to Fc receptors) and vascular endothelial cells (via pinocytosis) Once internalized, immunoglobulins are catabolized into amino acids within lysosomes.

Temporal lobe abscess is usually a result of otitis media that has spread to the _, while frontal lobe abscess is usually due to spread from _.

Temporal- mastoid air cells Frontal - ethmoid or frontal sinusitis A single brain abscess is usually caused by direct spread of a contiguous infection. Multiple brain abscesses typically indicate hematogenous dissemination of a distant infection (eg, endocarditis, empyema).

Cholinesterase inhibitors overcome this toxicity by inhibiting the degradation of acetylcholine, thereby increasing the concentration of acetylcholine at the synaptic cleft. Central nervous system (CNS) penetration and reversal of central symptoms are dependent on chemical structure:

Tertiary amines (eg, physostigmine, galantamine, donepezil, rivastigmine) are lipophilic (nonpolar) and can easily cross the blood-brain barrier to reverse both central and peripheral symptoms. Quaternary amines (eg, neostigmine, edrophonium, pyridostigmine) are hydrophilic (polarized) and do not readily cross the blood-brain barrier. These drugs reverse peripheral symptoms only

Thayer-Martin medium components

Thayer-Martin VCN selective medium contains vancomycin, colistin, nystatin, and trimethoprim. These antibiotics kill potential contaminants such as gram-positive organisms (vancomycin), gram-negative organisms other than Neisseria (colistin and trimethoprim), and fungi (nystatin).

Where is chemoreceptor trigger zone (CTZ) located

The CTZ is located in the area postrema in the medulla, located on the floor of the fourth ventricle. CTZ releases dopamine and serotonin, which in turn stimulates the medullary vomiting center. This induces nausea and vomiting. The CTZ lacks the the blood-brain barrier, which is critical to its role in detecting toxic substances in the circulation (including chemotherapy agents).

A prolonged course of total parenteral nutrition (TPN) is often complicated by gallstones. In normal individuals, enteral passage of fat and amino acids into the duodenum triggers release of cholecystokinin (CCK), leading to contraction of the gallbladder. The absence of normal enteral stimulation in patients receiving TPN leads to decreased CCK release and subsequent biliary stasis. In addition, patients with extensive resection of the ileum can have disruption to the normal enterohepatic circulation of bile acids, leading to inadequate solubilization of biliary cholesterol and formation of cholesterol crystals.

The absence of normal enteral stimulation in patients receiving total parenteral nutrition leads to decreased cholecystokinin release, biliary stasis, and increased risk of gallstones. Resection of the ileum can also increase the risk of gallstones due to disruption of normal enterohepatic circulation of bile acids.

Chagas disease is divided into 2 phases:

The acute phase (8-12 weeks after transmission) is characterized by circulating trypomastigotes that are detectable on blood microscopy. Most patients are asymptomatic, but a minority develop nonspecific symptoms (eg, fever, malaise, anorexia) and inflammation/swelling at the site of inoculation (eg, eye swelling following conjunctival inoculation). The chronic phase begins when the immune system eliminates circulating parasitemia, leading to resolution of acute symptoms. Patients subsequently develop a prolonged asymptomatic phase ("indeterminate form"), whereby serology is positive for T cruzi, but there are no symptoms, signs, detectable parasitemia, or indications of end-organ damage. After 1-3 decades, a minority of those with asymptomatic chronic infection develop end-organ disease of the heart or gastrointestinal system.

Vasospastic angina involves hyperreactivity of coronary artery smooth muscle. Patients are usually young (age <50) and without significant risk factors for coronary artery disease; they experience recurrent episodes of chest discomfort that typically occur during rest or sleep and resolve within 15 minutes.

The condition likely results from hyperreactivity of arterial smooth muscle due to a combination of endothelial dysfunction and autonomic imbalance

The major virulence factor of Hib is its polysaccharide capsule, which is composed of the polymer polyribosylribitol phosphate (PRP). The PRP capsule protects the bacterium against phagocytosis and complement-mediated lysis by binding factor H, a circulating regulator protein that normally prevents complement (C3b) deposition on host cells.

The conjugate Hib vaccine is composed of PRP conjugated to a protein toxoid. Immunization results in host development of anti-PRP antibody, which leads to improved opsonization and complement-mediated phagocytosis. Invasive Hib disease is rare in immunized patients, but it still occurs regularly in unimmunized children.

Chronic Chagas cardiomyopathy is relatively unique among DCMs in that there is characteristically localized apical wall thinning with the development of a large apical aneurysm. In addition to impairing ventricular systolic function, the aneurysm can harbor mural thrombus that may systemically embolize and cause stroke. Damage to the cardiac conduction system can also trigger ventricular arrhythmias (eg, ventricular tachycardia, ventricular fibrillation) that can lead to sudden cardiac death (the likely cause of death in this patient).

The gastrointestinal manifestations of Chagas disease result from destruction of the myenteric plexus, which can lead to progressive dilation and dysfunction of the esophagus and, less commonly, the colon.

Above the arcuate line, the rectus abdominis is surrounded by anterior and posterior sheaths; below, the muscle is covered only by the anterior sheath.

The inferior epigastric arteries below the arcuate line are susceptible to injury (eg, hematoma) due to lack of a supporting posterior rectus sheath

Medial pectoral (pectoralis minor and major) and lateral pectoral (pectoralis major) nerves: Damage results in weakness while adducting and medially rotating the arm. In addition, there may be weakness flexing (clavicular head) or extending (sternocostal head) the humerus

The intercostobrachial nerve is a purely sensory nerve that typically originates from the second intercostal nerve's lateral cutaneous branch and then traverses the axilla to innervate the skin of the axilla and medial upper arm. The intercostobrachial nerve is the most frequently injured nerve (whether by stretching or transection) during axillary lymph node dissection.

normal pressure hydrocephalus (NPH).

The lateral ventricles expand, resulting in normal intracranial pressure and stretching the descending cortical fibers (corona radiata). Interruption of these pathways, along with impairment of cortical and subcortical function, leads to the classic triad of NPH symptoms (dementia, gait disturbance, urinary incontinence). The cerebral cortex (through the pontine micturition center) regulates the micturition reflex, which is an autonomic spinal reflex mediated by the neurons in the sacral spinal cord. In NPH, the lack of cortical inhibition results in detrusor muscle hyperactivity. This leads to urinary urgency, which can be worsened by gait abnormalities that limit a patient's ability to quickly get to a bathroom (urge incontinence). Progressive frontal lobe impairment leads to the inability to recognize urinary urge, uncontrolled urination, and a lack of concern as the disease progresses.

The hemodynamic changes in AR are evident on cardiac catheterization and include reduced aortic diastolic pressure and elevated left ventricular (LV) diastolic pressure due to backflow of blood during diastole. In addition, aortic and LV systolic pressures are increased due to high stroke volume and a compensatory increase in cardiac contractility

The left ventricle becomes dilated with an increase in end-diastolic volume and undergoes eccentric hypertrophy that allows for a compensatory increase in stroke volume and cardiac output. The result of these effects is a characteristic high-amplitude, rapid rise-rapid fall pulsation with each ventricular contraction that manifests as widened pulse pressure, rapid distension and collapse of the carotid arteries (Corrigan sign), and a "to-and-fro" bruit appreciated over the femoral arteries (Duroziez sign).

Cleft lip

The lip and palate form during the fifth-sixth week of embryonic development through a series of fusions: The first pharyngeal arch splits into the upper maxillary prominence and the lower mandibular prominence. Fusion of the 2 medial nasal prominences forms the midline intermaxillary segment. The intermaxillary segment will become the philtrum of the upper lip, the 4 medial maxillary teeth, and the primary palate. The left and right maxillary prominences then fuse with the midline intermaxillary segment to form the upper lip and primary palate. If one of the maxillary prominences fails to fuse with the intermaxillary segment, a unilateral cleft lip results. If both maxillary prominences fail to fuse with the intermaxillary segment, bilateral cleft lip results.

Although rare, opsoclonus-myoclonus is a paraneoplastic syndrome highly associated with neuroblastoma and is believed to be an autoantibody response to central nervous system antigens.

The majority of patients with neuroblastoma have elevated catecholamine metabolites (eg, HVA, VMA) due to the neural crest origin. Biopsy of the mass is diagnostic and will reveal small round blue cells and Homer Wright rosettes.

As a class, all benzodiazepines should be excluded from use in conjunction with alcohol, barbiturates, neuroleptics, or 1st generation antihistamines.

The most common side effect of diazepam is sedation. Diazepam may also impair coordination and balance (sometimes avoided in the elderly for fear of falls), decrease memory and concentration, and produce confusion. These effects are amplified by the co-administration of other CNS-depressants

The pO2 in the left atrium and ventricle is lower than that in the pulmonary capillaries due to mixing of oxygenated blood from the pulmonary veins with deoxygenated blood from the bronchial circulation and thebesian veins

The normal A-a gradient results from shunting of {{c1::thebesian}} and {{c1::bronchial}} veins

The symptoms of cauda equina syndrome result from disruption of nerve roots to the sciatic nerve (eg, lower extremity weakness, radicular low back pain), pudendal nerve (eg, saddle anesthesia), and/or pelvic splanchnic nerves.

The pelvic splanchnic nerves (S2-S4) provide parasympathetic innervation to the hindgut, bladder, and urinary sphincters that promotes peristalsis, bladder emptying, and pelvic floor relaxation during defecation. Constipation and difficulty urinating are prominent symptoms in cauda equina syndrome due to loss of parasympathetic input to the bowel and bladder.

Sensation to the parietal pleura, chest wall, and skin of the chest is supplied by the intercostal nerves that derive from the ventral nerve rami

The phrenic nerve, which is derived from the C3-C5 nerve roots, delivers motor innervation to the diaphragm and carries pain fibers from the diaphragmatic and mediastinal pleura (not the costal pleura). Pain due to diaphragmatic and mediastinal pleura is often referred to the C3-C5 distribution at the base of the neck and over the shoulder.

Reticulocytes contain bluish cytoplasm and reticular precipitates of residual ribosomal RNA.

The ribosomal RNA appears blue microscopically after the application of the Wright-Giemsa stain.

Angiotensin II is also a potent vasoconstrictor that ultimately increases systemic vascular resistance and arterial pressure.

This patient developed acute kidney injury after undergoing percutaneous catheterization, a procedure that uses contrast material to evaluate the patency of the coronary arteries. In a patient with normal vital signs, this presentation suggests contrast-induced nephropathy (CIN). Patients with CIN typically have an acute rise in creatinine and blood urea nitrogen within 24-48 hours of contrast administration, followed by a gradual return to baseline. The etiology of CIN remains unclear but is likely multifactorial and includes: Direct cytotoxicity causing acute tubular necrosis, resulting in diffuse necrosis of the proximal tubular cells visible on histologic specimens and muddy brown casts on urinalysis Renal vasoconstriction causing medullary ischemia

Primary nocturnal enuresis (ie, bed-wetting at age ≥5 without prior nighttime urinary continence) is caused primarily by a brain maturational delay in the development of bladder control

This process of developing bladder control involves the following: Awareness of bladder filling Suppression of bladder contractions by the cerebral cortex Coordination of sphincter/detrusor function in the pontine micturition center This process, and therefore toilet training, is typically complete by age 4 in most children. However, nighttime bladder control may be delayed until school age in others, leading to primary nocturnal enuresis.

elderly patients are at particularly high risk of developing heat-related illness due to several features of normal aging that impair this response, including:

Tonic contraction of the peripheral vasculature, which limits heat transfer to the skin Reduced sweat gland density, which limits the ability to dissipate heat via evaporation Loss of rete pegs and dermal capillaries, which reduces the effective epidermal area available for heat transfer

a genetic disorder resulting in the abnormal development of the first and second pharyngeal arches.

Treacher-Collins syndrome (TCS) The craniofacial abnormalities (eg, mandibular, maxillary, and zygomatic bone hypoplasia) often result in airway compromise and feeding difficulties. In addition, absent or abnormal ossicles (eg, incus, malleus, stapes) lead to profound conductive hearing loss.

Patiromer MOA

Treatment for chronic hyperkalemia a nonabsorbable cation exchange resin that binds colonic potassium in exchange for calcium, trapping potassium within the resin where it is then excreted in the feces. It is often used for treatment of chronic hyperkalemia. However, onset of action takes several hours, so it is not recommended as monotherapy in acute hyperkalemia. Adverse effects include gastrointestinal disturbance (eg, diarrhea), hypokalemia, hypercalcemia (due to luminal exchange of calcium), and hypomagnesemia (due to off-target binding of other positive ions). Patiromer may also bind certain medications (eg, ciprofloxacin, levothyroxine). Sodium zirconium cyclosilicate is another nonabsorbable cation exchange resin that binds intestinal potassium in exchange for sodium and hydrogen. It is more selective for potassium and does not interfere with absorption of magnesium or other medications. However, the increased sodium load may be problematic for patients sensitive to exogenous sodium (eg, cirrhosis, congestive heart failure).

Describe findings of the paraneoplastic syndrome called Trousseau syndrome.

Trousseau syndrome is marked by symptoms of vessel inflammation caused by blood clots (thrombophlebitis, experienced by the patient as tender, superficial cords or nodules), which are recurrent or appear in different locations Trousseau syndrome is a unique migratory thrombophlebitis that occurs as a result of the liberation of clotting factors, such as tissue factor, by some pancreatic tumors. The presence of thrombophlebitis often correlates with advanced malignancy and is a leading cause of cancer-related deaths

Epididymitis caused by infection (eg, Neisseria gonorrhoeae) or trauma causes acute scrotal pain that is relieved with manual elevation of the testicle

Varicocele, which is characterized by a scrotal mass with a "bag of worms" texture, increases in size when standing as compared with supine positioning Increased abdominal pressure (eg, cough, Valsalva maneuver) can lead to bulging within the groin or scrotum due to an inguinal hernia.

Neuroinvasive infection is an uncommon complication of WNV but has a 10% fatality rate. It occurs most commonly in older patients and those with a history of malignancy or organ transplant, and is characterized by meningitis (ie, headache, meningismus) and/or encephalitis (altered mental status). The presence of acute-onset asymmetric flaccid paralysis is highly suggestive of WNV, particularly if the patient demonstrates concurrent parkinsonian features (eg, rigidity, bradykinesia, tremor, postural instability). Diagnosis can be made when compatible clinical findings are associated with positive cerebrospinal fluid anti-WNV antibodies (polymerase chain reaction testing often not needed).

West Nile virus (WNV) infection

Neonatal vitamin K deficiency is prevented by administration of intramuscular vitamin K at birth.

Without exogenous vitamin K, as in this patient, vitamin K deficiency results in impaired clotting factor carboxylation and, therefore, a propensity for bleeding. Intracranial hemorrhage (ICH) is a potentially fatal complication, often presenting with signs of increased intracranial pressure (eg, altered mental status, enlarging head circumference, bulging fontanel, downward-driven eyes).

Focal-onset seizures most commonly occur due to

a focal structural abnormality (eg, stroke, neoplasm) that causes a region of hyperexcitable neuronal activity in one cerebral hemisphere. These seizures begin with localized symptomatology (eg, muscle twitching, paresthesia) and are classified according to level of consciousness (eg, aware, impaired awareness). They can progress to bilateral tonic-clonic seizures if the excitatory neuronal activity spreads to involve both cerebral hemispheres.

Describe Treacher-Collins syndrome (TCS)

a genetic disorder resulting in the abnormal development of the first and second pharyngeal arches. The craniofacial abnormalities (eg, mandibular, maxillary, and zygomatic bone hypoplasia) often result in airway compromise and feeding difficulties. In addition, absent or abnormal ossicles (eg, incus, malleus, stapes) lead to profound conductive hearing loss.

Symptomatic management of peripheral artery disease (PAD) includes

a graded exercise program and cilostazol. Cilostazol is a phosphodiesterase inhibitor that inhibits platelet aggregation and acts as a direct arterial vasodilator. Patients with PAD should also receive an antiplatelet agent (aspirin or clopidogrel) for secondary prevention of coronary heart disease and stroke

Mastocytosisis =

a group of disorders that involves an increased number of mast cells at the skin (dermis) or other organs * Cutaneous mastocytosis: limited to skin * Systemic mastocytosis: multiorgan involvement (e.g., bone marrow, GI tract) with or without skin manifestations * Associated with mutations in c-Kit gene * Results in high histamine levels, leading to: * Pruritus, flushing, diarrhea * Gastric ulcers (due to increased gastric acid secretion)

Describe RNA interference

a mechanism by which short (20-30 base pair) double-stranded RNA sequences induce posttranscriptional gene silencing. After being transcribed, miRNA undergoes processing in the nucleus to form a double-stranded precursor that is then exported into the cytoplasm. There, the precursor is cleaved into a short RNA helix by a ribonuclease protein called dicer. The individual strands are then separated and incorporated into RNA-induced silencing complex (RISC). This multiprotein complex uses its associated miRNA as a template to bind to complementary sequences found on target mRNAs. An exact match generally results in mRNA degradation, but a partial match also causes translational repression by preventing ribosome and translation factor binding.

Describe Bronchophony

a phenomenon in which spoken words (eg, "ninety-nine," "toy boat") are heard more loudly and clearly when auscultated over an area of alveolar consolidation (eg, lobar pneumonia) compared to normal lung. It results from increased sound transmission due to increased tissue density.

Cilostazol MA

a phosphodiesterase inhibitor that inhibits platelet aggregation and acts as a direct arterial vasodilator. Patients with PAD should also receive an antiplatelet agent (aspirin or clopidogrel) for secondary prevention of coronary heart disease and stroke Use for Symptomatic management of peripheral artery disease (PAD)

Neuropeptide Y (NPY)

a polypeptide neurotransmitter found in the central and autonomic nervous systems. It is thought to play a role in appetite and pain perception. NPY antagonists have been investigated as potential antiobesity drugs.

Hantavirus presents with

a severe, sudden onset of pulmonary edema. The virus is found in North, Central, and South America; infections in the United States are most commonly observed in the Southwest. Rodents are the natural reservoir for hantavirus; the disease is spread through contact with the feces of deer mice.

In cases of suspected erythroblastosis fetalis, amniocentesis is used to assess fetal hemolysis by measuring

amniotic fluid bilirubin levels.

Titin function

an elastic protein that anchors the beta-myosin heavy chain to the Z-discs and likely contributes to passive myocardial tension; absence of complete titin proteins leads to myocardial dysfunction. TTN gene mutations (familial dilated cardiomyopathy) follow autosomal dominant inheritance; however, they have incomplete penetrance, leading to delayed or absent clinical manifestations in some family members.

intermittent explosive disorder (IED)

an impulse control disorder. Patients with IED often feel a sense of heighted arousal or tension that quickly escalates to rage and uncontrollable impulses to be verbally or physically aggressive. Outbursts may provide an immediate sense of relief that is usually followed by remorse. Patients can be assaultive and destroy property, resulting in significant functional impairment due to interpersonal difficulties, school suspension, job loss, and legal problems.

Elevated eosinophils contribute to the host defense against parasites through what mechanism?

antibody-dependent cell-mediated cytotoxicity (ADCC) (which is also used by macrophages, neutrophils, and natural killer cells) Eosinophil proliferation and activation during multicellular parasitic infection is stimulated by IL-5 produced by TH2 and mast cells (not to be confused with IL-4, which stimulates IgE production). When a parasite invades the mucosa or enters the bloodstream, it is coated by IgG and IgA antibodies that bind the Fc receptors located on the eosinophil cell surface. This triggers eosinophil degranulation and release of cytotoxic proteins (eg, major basic protein) and reactive oxygen intermediates, substances that damage and destroy antibody-bound parasites.

Isolated systolic hypertension (systolic blood pressure >140 mm Hg with diastolic blood pressure <90 mm Hg) is due to age-related stiffness and decrease in compliance of the aorta and major peripheral arteries

arbonate concentration increases. involves a total lack of communication between the atria and ventricles due to AV node dysfunction. It can result from ischemia, infiltrative disease (eg, sarcoidosis), infection (eg, Lyme disease), or age-related fibrosis with cellular degeneration. Because conduction signals from the atria cannot communicate with the ventricles, the intrinsic pacemaker of the His bundle or ventricles is triggered, resulting in a junctional or ventricular escape rhythm On ECG, there is dissociation of P waves and QRS complexes, with P waves marching out at the intrinsic rate of the sinoatrial node and QRS complexes at the intrinsic rate of the His bundle or ventricles (escape rhythm).

Deficiency of vitamin A can develop in patients with

biliary disorders, exocrine pancreatic insufficiency, or intestinal malabsorption.

Hemicholinium MOA

blocks choline uptake by the presynaptic neuron, preventing ACh synthesis through depletion of intracellular choline.

In the management of septic shock, rapid restoration of intravascular volume and adequate end-organ perfusion is critical. This is best accomplished with the administration of

boluses of isotonic crystalloid in the form of 0.9% (normal) saline or lactated Ringer solution through large-bore, peripheral intravenous catheters. These solutions are ideal for volume resuscitation because they have osmolarity very close to the normal osmolarity of the blood (~285 mOsm/kg H2O), while the sodium and chloride ions help retain the fluid in the extracellular space.

The development of AF most commonly involves ectopic electrical foci in the pulmonary veins that trigger fibrillatory conduction in abnormal (remodeled) atrial tissue.

catheter ablation of pulmonary vein trigger sites (pulmonary vein isolation) is used for the treatment of symptomatic, paroxysmal AF.

What is subcutaneous crepitus

crackling during palpation See in pneumothorax Disruption of the skin or parietal pleura during the initial injury often allows air to enter the subcutaneous tissues of the chest wall, which is detected as subcutaneous crepitus

Dengue viruses are single-stranded RNA viruses with 4 different serotypes. Primary (first) infection can be asymptomatic or cause a self-limited disease in most adults. Secondary infection is due to infection with a different viral serotype and usually causes more severe illness

dengue hemorrhagic fever (DHF). Dengue viruses are transmitted by the Aedes mosquito and are single-stranded RNA viruses (genus Flavivirus) with 4 different serotypes (DENV1-4). Primary (first) infection can be asymptomatic or cause a self-limited disease in most adults. Symptomatic disease can cause high-grade fever, headache, retro-orbital pain, bleeding (eg, epistaxis, petechiae/purpura), diffuse macular rash, muscle and joint pain ("break-bone fever"), leukopenia, thrombocytopenia, and elevated liver enzymes. Milrinone is a phosphodiesterase-3 inhibitor that reduces the degradation of cyclic adenosine monophosphate to provide 2 beneficial effects for treating systolic heart failure. Calcium influx into cardiomyocytes is increased, which increases cardiac contractility. In addition, calcium-myosin light chain kinase interaction is reduced, which causes vasodilation and reduces cardiac preload and afterload

When see low T3 level, normal thyroid-stimulating hormone (TSH), and normal T4

euthyroid sick syndrome (ESS) (also known as low T3 syndrome)

In cognitive testing; asking the patient to draw a clock oriented to the time requested tests what function

executive function

Why is Listeria monocytogenes resistant to third-generation cephalosporins

has intrinsic resistance to third-generation cephalosporins (eg, cefotaxime, ceftriaxone) due to the presence of penicillin-binding proteins that have a low-affinity for this drug class. Coverage of Listeria requires the addition of ampicillin, a beta-lactam antibiotic with a specialized side-group that allows it to bind to and inactivate the penicillin binding proteins of Listeria.

VHL encodes a protein that normally inhibits what

hypoxia-inducible factors; VHL mutations lead to constitutive activation of these proteins, resulting in overexpression of multiple angiogenic growth factors (eg, VEG-F, PDG-F) that stimulate tumor proliferation. As a result, tumors harboring VHL mutations are highly sensitive to angiogenesis inhibitors.

With moderate to heavy exercise, the changes to the left ventricular pressure-volume loop include

increased preload and increased contractility that both contribute to increased stroke volume. Although reduced systemic vascular resistance decreases afterload, the increase in systolic pressure caused by increased preload and contractility override the decrease, resulting in overall increased afterload.

Bromoacetylcholine MOA

inhibits choline acetyltransferase, blocking the synthesis of ACh from acetate and choline.

Fractures to the orbital floor commonly result from direct frontal trauma to the orbit. The infraorbital nerve runs along the orbital floor in a groove in the maxilla before exiting the skull just inferior to the orbit. Damage can result in paresthesia of the upper cheek, upper lip, and upper gingiva. In addition, the inferior rectus muscle can also become entrapped, limiting vertical gaze.

injuries. The infraorbital nerve (continuation of the maxillary nerve) runs along the orbital surface of the maxilla in the infraorbital groove before traversing the infraorbital canal and exiting the skull via the infraorbital foramen (just below the orbit).

the deep branch of radial nerve innervates

innervates the extensor compartment muscles in the forearm.

Blood circulates in the kidney along the following path: _ arteries —> etc.

interlobar arteries→arcuate arteries→radial arteries→afferent arterioles→glomerular capillaries→efferent arterioles→peritubular capillaries→interlobar veins.

Damage to the _ can occur when foreign bodies (eg, chicken or fish bones) become lodged in the piriform recess or during attempts to retrieve them. The resulting reduction in laryngeal sensation can impair the _ reflex

internal laryngeal nerve (branch of superior laryngeal which is branch of vagus) cough reflex.

The deep cervical fascia is composed of three layers:

investing, pretracheal, and prevertebral. The prevertebral layer of the deep fascia surrounds the vertebral column and spinal muscles

Describe A case series study

is a descriptive study that tracks patients with a known condition (eg, a particular exposure, risk factor, or disease) to document the natural history or response to treatment. Unlike a case-control study, a case series is a descriptive study that cannot quantify statistical significance.

Opioid withdrawal can appear similar to other withdrawal or intoxication syndromes, but certain findings, including_ are fairly specific.

lacrimation and yawning Opioid withdrawal can start as soon as 6-12 hours after cessation of a short-acting opioid such as heroin and usually peaks within 24-48 hours. Symptoms include nausea, vomiting, diarrhea, abdominal cramping, and myalgia. It is generally nonlife-threatening and vital signs are usually normal. Dilated pupils, prominent piloerection, and hyperactive bowel sounds are seen on examination.

The 2 most important factors for osteoclastic differentiation are

macrophage colony-stimulating factor (M-CSF) and receptor for activated nuclear factor kappa-B ligand (RANK-L), are produced by osteoblasts and bone marrow stromal cells.

What is released in trousseau syndrome

migratory thrombophlebitis that occurs as a result of the liberation of clotting factors, such as tissue factor, by some pancreatic tumors. Equalization of diastolic pressures between left ventricle and right ventricle occurs in cardiac tamponade.

What are arthropods

mosquitos, ticks, fleas

Autosomal dominant mutations in the TTN gene, which encodes for the sarcomere protein titin is involved in

most common cause of familial dilated cardiomyopathy

List causes of hypophosphatemia due to Internal redistribution

ncreased insulin secretion (especially refeeding malnourished patients) A cute respiratory alkalosis (stimulates glycolysis) Hungry bone syndrome (after parathyroidectomy)

Although patients with HD display generalized cortical atrophy as the disease progresses, the most prominent atrophy is seen bilaterally in the caudate nucleus and putamen, leading to the enlargement of the frontal horns of the lateral ventricles

ne is classically seen in common variable immunodeficiency syndrome (CVID), a disorder of low immunoglobulin and B lymphocyte levels that predisposes to recurrent sinopulmonary and gastrointestinal infections ere, ischemic pain at rest and possible distal gangrene.

This infant has pneumatosis intestinalis (ie, air in the bowel wall), which can be seen on abdominal x-ray as thin curvilinear areas of lucency that parallel the lumen. In an infant with abdominal distension and bloody stools, this finding is diagnostic for

necrotizing enterocolitis (NEC). Upon initiation of enteral feeding, bacteria are introduced into the bowel where they proliferate excessively due to compromised immune clearance. Impaired mucosal barrier function allows the bacteria to invade the bowel wall, causing inflammation and ischemic necrosis of the terminal ileum and colon. As the disease progresses, the bowel becomes congested and gangrenous with the formation of intramural gas collections. Up to 30% of affected neonates die, especially when disease is complicated by intestinal perforation. Survivors are at risk for strictures and bowel obstruction secondary to fibrosis that occurs as the inflammation subsides.

Nonselective beta blockers attenuate the norepinephrine/epinephrine-mediated symptoms of hypoglycemia, but cholinergic symptoms (eg, hunger) are unaffected. In addition, nonselective beta blockers can contribute to the severity of hypoglycemia; epinephrine stimulates hepatic gluconeogenesis and peripheral glycogenolysis, but this effect is diminished by nonselective beta blockers.

negative inotropic agents such as beta blockers (metoprolol), nondihydropyridine calcium channel blockers (verapamil), and disopyramide reduce LVOT obstruction and are helpful in symptomatic patients with HCM (Choices B, D, and E). In addition, beta blockers may also help reduce anginal symptoms by decreasing myocardial oxygen demand.

RNA molecules that carry out functions without first being translated into proteins are called

non-coding RNAs. These include small nuclear RNA (snRNA), ribosomal RNA (rRNA), and transfer RNA (tRNA).

Vesamicol MOA

noncompetitively blocks the vesicular ACh transporter, preventing ACh from entering secretory vesicles in the presynaptic nerve terminal.

Methemoglobinemia is characterized by a high blood level of methemoglobin, in which the oxygen-carrying iron is present in the Fe3+ or Fe2+ state?

oxidized (ferric Fe3+) state instead of the reduced (ferrous Fe2+) state. Methemoglobin cannot bind to oxygen. Moreover, any remaining ferrous hemes in the hemoglobin tetramer exhibit increased oxygen affinity (ie, the oxygen-hemoglobin dissociation curve is shifted to the left). Thus, signs and symptoms of methemoglobinemia reflect decreased blood oxygen content and an even greater decrease in oxygen delivery to tissues (ie, cellular hypoxia). Additional symptoms include headache, nausea, confusion, seizures, and coma. Because oxygen diffusion at the alveolar-arterial level is not impaired, the arterial partial pressure of oxygen is normal. Blood may have a characteristic muddy color, secondary to the oxidization state of iron. Methemoglobinemia may occur as an adverse effect of oxidizing agents such as sulfonamides, dapsone, and local anesthetics (eg, benzocaine), from hereditary hemoglobin abnormalities, or secondary to a hereditary deficiency of the reduced form of nicotinamide adenine dinucleotide.

abdominal x-ray as thin curvilinear areas of lucency that parallel the lumen in an infant

pneumatosis intestinalis (ie, air in the bowel wall) In an infant with abdominal distension and bloody stools, this finding is diagnostic for necrotizing enterocolitis (NEC).

What innervates muscles involved in finger and thumb extension

posterior interosseous nerve (After passing between the superficial and deep parts of the supinator muscle, the deep branch of radial nerve continues as the posterior interosseous nerve)

Arginase deficiency results in

progressive spastic diplegia, growth delay, and abnormal movements. Treatment includes an arginine-free, low-protein diet. Arginase is a urea cycle enzyme that produces urea and ornithine from arginine.

The median nerve gains access to the forearm in the medial aspect of the antecubital fossa and then immediately courses between the humeral / ulnar heads of the _ muscle

pronator teres muscle ompression by the pronator teres occurs proximal to the takeoff of the palmar branch; therefore, sensation is impaired not only to the palmar aspect of the first 3½ digits but also to the entire lateral palm and thenar eminence (ie, sensory distribution of the palmar branch). (vs. when in carpal tunnel)

The superficial branch of radial nerve innervates

provides purely somatic sensory innervation to the radial half of the dorsal hand

Isoniazid is chemically similar to what molecule

pyridoxine (vitamin B6). Because of this similarity, isoniazid can compete with vitamin B6 in the synthesis of multiple neurotransmitters (including GABA), resulting in defective end products. Isoniazid also increases the urinary excretion of pyridoxine, and a frank deficiency of this vitamin can eventually develop

Sudden-onset abdominal or flank pain, hematuria, and left-sided varicoceles suggest what complication of nephrotic syndrome

renal vein thrombosis, a well-known complication of nephrotic syndrome. Nephrotic syndrome is a hypercoagulable state. Loss of anticoagulant factors, especially antithrombin III, is responsible for the thrombotic and thromboembolic complications of nephrotic syndrome.

Crescents consist of glomerular parietal cells, lymphocytes, and macrophages along with abundant fibrin deposition. Crescents eventually become fibrotic, disrupting glomerular function and causing irreversible renal injury

resulting in an influx of coagulation factors (eg, fibrinogen) and inflammatory cells (eg, lymphocytes, macrophages) into Bowman's space. Initiation of the coagulation cascade promotes the deposition of large quantities of fibrin, while inflammatory cells proliferate and release growth factors and inflammatory cytokines that recruit fibroblasts and stimulate parietal cell proliferation. This eventually results in progressive glomerular hypercellularity, fibrosis, and irreversible renal injury

Effect modification

results when an external variable positively or negatively impacts the observed effect of a risk factor on disease status. When analysis is performed based on stratification by this external variable, there will be a significant difference in risk between the stratified groups. For instance, aspirin use is associated with Reye syndrome in children but not adults; therefore, age modifies the effect of aspirin on Reye syndrome development. Effect modification can easily be confused with confounding; but stratified analysis can help distinguish between them. With confounding, there is usually no significant difference between the strata

Irreversible _ can occur with long-term use of hydroxychloroquine.

retinal damage

6-MP is metabolized into inactive metabolites by 2 different enzymes, xanthine oxidase and thiopurine methyltransferase (TPMT)

roughly 12% of the population has a genetic mutation resulting in low or absent activity of TPMT, creating a high risk of toxicity with the administration of 6-MP. Therefore, some clinicians recommend that patients undergo genetic testing for TPMT activity prior to initiating 6-MP (or azathioprine) therapy.

Why not see Schistosoma haematobium in United States

schistosomiasis caused by Schistosoma haematobium. Humans acquire schistosomiasis via contact with infected freshwater. Infection is not possible in the United States due to absence of the specific freshwater snails that incubate the infectious larvae

Epstein-Barr virus (EBV) commonly infects B cells, stimulating them to proliferate continuously ("transformation" or "immortalization"). EBV is an oncogenic virus that promotes polyclonal B cell proliferation and heterophile antibody production. The heterophile antibody test is sensitive and highly specific for EBV infection.

simple myopia (nearsightedness), a refractive error caused by an increased anterior-posterior diameter of the eye. The risk is greatest for those with a positive family history or those of East Asian descent. During the adolescent growth spurt, the axial length of the globe also grows rapidly, making the incidence of new-onset myopia highest in this age group. As the anterior-posterior diameter increases, the focal point of a refracted image lies anterior to the retina. For images to be seen clearly, they must be projected directly onto the retina; therefore, the image displacement in myopia causes blurred vision of long-distance objects and decreased visual acuity on examination. Vision of nearby objects is unaffected and appears clear in myopic patients.

Arboviruses =

small RNA viruses that are harbored by birds and small mammals and are transmitted to humans via biting arthropods (eg, mosquitos, ticks, fleas), primarily during the summer months when arthropod populations are at their peak.

NADPH is used in anabolic processes (eg,

steroid and fatty acid synthesis) as a supply of reducing equivalents.

After passing between the superficial and deep parts of the supinator muscle, the deep branch of radial nerve continues as

the posterior interosseous nerve, which innervates muscles involved in finger and thumb extension.

What occurs to the normal/nonstenotic kidney in unilateral RAS

the contralateral, nonstenotic kidney is exposed to high blood pressure and therefore demonstrates typical signs of hypertensive nephrosclerosis, which is characterized by arteriolar wall thickening due to hyaline arteriolosclerosis (hyalinization of the arterioles due to extravasation of plasma proteins) and hyperplastic arteriolosclerosis (concentric smooth muscle cell proliferation in response to pressure ["onion-skinning"]).

What is diaphragmatic excursion and what is it in pneumothorax

the distance the diaphragm moves between expiration and inspiration The increased thoracic pressure with pneumothorax flattens the resting position of the diaphragm and restricts its movement with respiration. Therefore, diaphragmatic excursion is decreased

The ELEK test

the immunodiffusion test used to identify toxin-producing strains of Corynebacterium diphtheriae.

The thoracic duct drains into the junction between

the left subclavian and jugular veins in the lower neck The thoracic duct is the largest lymphatic vessel. It originates at about the level of T12 in the abdomen where the lumbar and intestinal lymph trunks converge. It travels superiorly through the mediastinum (posterior to the esophagus) and drains into the junction between the left subclavian and jugular veins in the lower neck. It is most commonly injured during thoracic procedures (eg, esophagectomy) but can be injured in neck procedures as well (eg, cervical lymph node removal). Injury can result in leakage of lymph into either the neck or the thorax because a transected duct may retract into the chest.

Once released into the environment, the larvae can penetrate intact human skin and subsequently travel to the liver where they mature into adults over a period of several weeks. After maturation, the adult worms migrate to specific destinations:

the mesenteric venules of the intestine (S japonicum and S mansoni) or the urinary bladder venous plexus (S haematobium). The adult worms remain in these blood vessels for life (5-30 years), adhering to the vessel wall with suckers and releasing eggs into circulation. Eggs released by S japonicum and S mansoni have a tendency to penetrate the bowel wall and be excreted in the feces. They also frequently traverse the portal venous system and lodge in the liver. S haematobium eggs tend to pierce the vesical and ureteral walls and be expelled in the urine. On exposure to freshwater, these eggs release larvae that can infect snails and perpetuate the life cycle.

The abnormal red cells of sickle-cell anemia can sickle within the area of high osmolarity, hypoxia, and slow blood flow rates of what blood vessels in kidney

the vasa recta of the renal medulla. Sickling within the vasa recta can lead to a patchy papillary necrosis, hematuria, proteinuria, and sometimes renal scarring

vaccines containing the polysaccharide antigen alone are ineffective in children age <2 years due to

their immature humoral immunity. Bacteria with polysaccharide capsules (eg, Hib, Streptococcus pneumoniae [pneumococcus], Neisseria meningitidis) are antiphagocytic. The polysaccharide capsule provokes an antibody-mediated (B cell) immune response and is the primary antigenic constituent of vaccines against encapsulated bacteria. However, vaccines containing the polysaccharide antigen alone are ineffective in children age <2 years due to their immature humoral immunity. Therefore, the polysaccharide is conjugated with a carrier protein to amplify the patient's humoral response against the polysaccharide through T cell recruitment. The Hib conjugate vaccine contains a carrier protein that is derived from either a tetanus toxoid (TT) protein or an outer membrane protein (OMP) of Neisseria meningitidis. The polysaccharide-protein conjugate then becomes a T cell-dependent antigen. Immunogenicity is increased as a result of T cell-dependent stimulation of B lymphocytes and the production of memory B lymphocytes.

First-line treatment for Pediculus humanus capitis (head lice) includes

topical pediculicides (eg, permethrin, ivermectin). Manual lice removal via wet combing is an alternate treatment for patients for whom standard pediculicides are not feasible (eg, children age <2 years, resource-poor populations).

Expiratory stridor often occurs due to

tracheal pathology (eg, tracheomalacia) (vs. laryngeal for inspiratory) because inspiration decreases intrathoracic pressure (relative to the airways), which widens the intrathoracic trachea. Expiration increases intrathoracic pressure, which narrows the intrathoracic trachea, worsening the obstruction.

Osteoporosis due to hyperparathyroidism affects _ bone while that due to aging predominately affects _ bone

typical osteoporosis of aging predominantly affects trabecular bone, osteoporosis in PHPT is most pronounced in the cortical (compact) bone of the appendicular skeleton (eg, pectoral girdle, pelvic girdle, limbs).

lymph drainage of the esophagus:

upper 1/3: cervical nodes middle 1/3: mediastinal nodes lower 1/3: celiac and gastric nodes

What happens to RPF and GFR in severe efferent arteriolar constriction

when RPF is low secondary to severe efferent arteriolar constriction, the substantially increased capillary oncotic pressure results in an overall decrease in GFR Decrease GFR, Increase FF

RNA {{c1::interference}} uses {{c2::dsRNA}} complementary to an mRNA of interest to "knock down" gene expression

when transfected into human cells, dsRNA is separated (by dicer) and promotes degradation of target mRNA via RISC (RNA-induced silencing complex) an exact match generally results in mRNA degradation, but a partial match also causes translational repression by preventing ribosome and transcription factor binding

stridor is classically worse in __ position, improves with _ position

worse in supine position, improves with upright position patients with airway obstruction at the level of the larynx (eg, epiglottitis, laryngomalacia, bilateral vocal cord paralysis) would be expected to have inspiratory stridor.


Kaugnay na mga set ng pag-aaral

Money and Banking Chapter 5 Questions

View Set

Chapter 47: Caring for Clients with Disorders of the Liver, Gallbladder, or Pancreas

View Set

W8 - Unemployment & Economic Growth (Theory)

View Set

Investments in Debt & Equity Securities

View Set

MGNT - 4080 Special Topics: Project Management

View Set

Module 48: Social-Cognitive Theories and the self

View Set

Seeing the Principles in our Government today: Each of the following phrases describes a situation dealing with one of the Principles of the Constitution. Next to each statement, state which Principle it relates to.

View Set